repositorio.unesum.edu.ecrepositorio.unesum.edu.ec/bitstream/53000/2123/1/LIBRO-DIP-039.pdf ·...

122

Transcript of repositorio.unesum.edu.ecrepositorio.unesum.edu.ec/bitstream/53000/2123/1/LIBRO-DIP-039.pdf ·...

Page 1: repositorio.unesum.edu.ecrepositorio.unesum.edu.ec/bitstream/53000/2123/1/LIBRO-DIP-039.pdf · Quedan todos los derechos reservados. Esta publicación no puede ser reproducida, distribuida,
Page 2: repositorio.unesum.edu.ecrepositorio.unesum.edu.ec/bitstream/53000/2123/1/LIBRO-DIP-039.pdf · Quedan todos los derechos reservados. Esta publicación no puede ser reproducida, distribuida,
Page 3: repositorio.unesum.edu.ecrepositorio.unesum.edu.ec/bitstream/53000/2123/1/LIBRO-DIP-039.pdf · Quedan todos los derechos reservados. Esta publicación no puede ser reproducida, distribuida,

ELEMENTOS DE MATEMÁTICA BÁSICA PARA CARRERAS UNIVERSITARIAS

TOMO II

Dr. C. ALBERTO RODRÍGUEZ RODRÍGUEZ. PhD.

Dr. C. JULIO C. PINO TARRAGÓ. PhD.

Ing. MARIBEL C. VÁSQUEZ PAUCAR. Mg.

Ing. MARCELA PINCAY PILAY. Mg.

Ing. MILTON CAÑARTE ÁVILA. Mg.

Ing. ÁNGEL PISCO GÓMEZ. Mg.

Ing. JIMMY GUTIÉRREZ GARCÍA. Mg.

Page 4: repositorio.unesum.edu.ecrepositorio.unesum.edu.ec/bitstream/53000/2123/1/LIBRO-DIP-039.pdf · Quedan todos los derechos reservados. Esta publicación no puede ser reproducida, distribuida,

Quedan todos los derechos reservados. Esta publicación no puede ser reproducida, distribuida, comunicada públicamente o utilizada, total o parcialmente, sin previa autorización.

© del texto: los autores

ÁREA DE INNOVACIÓN Y DESARROLLO, S.L.

C/ Els Alzamora, 17 - 03802 - ALCOY (ALICANTE) [email protected]

Primera edición: enero 2018

ISBN: 978-84-948074-2-8

DOI: http://dx.doi.org/10.17993/CcyLl.2018.13

Editorial Área de Innovación y Desarrollo,S.L.

Page 5: repositorio.unesum.edu.ecrepositorio.unesum.edu.ec/bitstream/53000/2123/1/LIBRO-DIP-039.pdf · Quedan todos los derechos reservados. Esta publicación no puede ser reproducida, distribuida,
Page 6: repositorio.unesum.edu.ecrepositorio.unesum.edu.ec/bitstream/53000/2123/1/LIBRO-DIP-039.pdf · Quedan todos los derechos reservados. Esta publicación no puede ser reproducida, distribuida,
Page 7: repositorio.unesum.edu.ecrepositorio.unesum.edu.ec/bitstream/53000/2123/1/LIBRO-DIP-039.pdf · Quedan todos los derechos reservados. Esta publicación no puede ser reproducida, distribuida,

Índice

CAPÍTULO I. LÓGICA MATEMÁTICA ................................................................................................................... 13

1.1. INTRODUCCIÓN ....................................................................................................................................... 13 1.1.1 OBJETIVOS ........................................................................................................................................... 13 1.1.2 Conceptos Fundamentales .................................................................................................................. 13 1.1.3 Desarrollo de los conceptos fundamentales ........................................................................................ 14 1.1.4 Ejemplificando ..................................................................................................................................... 14 1.1.5 Respuestas ........................................................................................................................................... 14

1.2. TIPOS DE PROPOSICIONES ............................................................................................................................... 14 1.3. OPERADORES LÓGICOS: (LLAMADOS TAMBIÉN CONECTORES LÓGICOS) SON PALABRAS QUE SIRVEN PARA ENLAZAR

PROPOSICIONES SIMPLES O CAMBIAR EL VALOR DE VERDAD DE UNA PROPOSICIÓN ................................................................... 15 1.4. TABLAS DE VALORES DE VERDAD ...................................................................................................................... 16 1.5. FUNCIONES Y POLINOMIOS BOOLEANOS. ........................................................................................................... 20 1.6. FUNCIONES BOOLEANAS ................................................................................................................................ 22 1.7. FUNCIONES CANÓNICAS MINTÉRMINOS Y MAXTÉRMINOS Y SUS CIRCUITOS LÓGICOS CORRESPONDIENTES EN SERIES Y EN

PARALELO. ................................................................................................................................................................ 23 1.8. MINI TÉRMINOS ........................................................................................................................................... 24

1.8.1 Forma canónica "suma de mini término"……………………………………………………………………………………..22

1.9. CONSTRUCCIÓN ALGEBRAICA ........................................................................................................................... 25

1.9.1 Maxi Términos……………………………………………………………………………………………………………….……………….23

1.10. FORMA CANÓNICA “PRODUCTO DE MAXITÉRMINOS” ........................................................................................... 26 1.11. APLICACIONES .............................................................................................................................................. 29

CAPÍTULO II GEOMETRÍA ANALÍTICA DEL PLANO ............................................................................................... 39

2.1 INTRODUCCIÓN ............................................................................................................................................ 39 2.2 GEOMETRÍA PLANA ....................................................................................................................................... 40 2.3 FÓRMULAS BÁSICAS: ..................................................................................................................................... 40 2.4 INTERSECCIÓN ENTRE RECTAS Y CURVAS ............................................................................................................. 41 2.5 EJEMPLOS ................................................................................................................................................... 41 2.6 ECUACIONES Y CARACTERÍSTICAS DE LA CIRCUNFERENCIA ...................................................................................... 43 2.7 EJEMPLOS ................................................................................................................................................... 43 2.8 FORO. - EJERCICIO PROPUESTOS ...................................................................................................................... 44 2.9 ECUACIONES Y CARACTERÍSTICAS DE LA PARÁBOLA ............................................................................................... 45 2.10 ECUACIÓN CANÓNICA DE LA PARÁBOLA ............................................................................................................. 46 2.11 ECUACIÓN DE LA PARÁBOLA DE VÉRTICE V(H;K) .................................................................................................. 47 2.12 ECUACIONES Y CARACTERÍSTICAS DE LA ELIPSE..................................................................................................... 48 2.13 ECUACIONES Y CARACTERÍSTICAS DE LA HIPÉRBOLA .............................................................................................. 49 2.14 APLICACIÓN ................................................................................................................................................. 52 2.15 EJERCICIOS RESUELTOS ................................................................................................................................... 52 2.16 EVALUACIÓN DE LA SECCIÓN ........................................................................................................................... 53 2.17 EJERCICIOS SOBRE CIRCUNFERENCIA .................................................................................................................. 55 2.18 EJERCICIOS SOBRE LA ECUACIÓN DE LA PARÁBOLA ................................................................................................ 56 2.19 EJERCICIOS SOBRE ELIPSE ................................................................................................................................ 56 2.20 EJERCICIOS SOBRE HIPÉRBOLA .......................................................................................................................... 57 2.21 EJERCICIOS INTEGRADORES DE LAS DIFERENTES CURVAS DE SEGUNDO GRADO ............................................................ 58

CAPÍTULO III. NÚMEROS COMPLEJOS ................................................................................................................ 61

3.1 ¿QUÉ SON LOS NÚMEROS COMPLEJOS? .................................................................................................. 61 3.2 DEFINICIÓN DE NÚMEROS COMPLEJOS .............................................................................................................. 62 3.3 FORMA TRIGONOMÉTRICA DEL NÚMERO COMPLEJO ............................................................................................. 63 3.4 DEFINICIÓN DOS NÚMEROS COMPLEJOS ............................................................................................................ 65 3.5 ADICIÓN DE NÚMEROS COMPLEJOS .................................................................................................................. 65 3.6 SUSTRACCIÓN DE NÚMEROS COMPLEJOS ........................................................................................................... 65

Page 8: repositorio.unesum.edu.ecrepositorio.unesum.edu.ec/bitstream/53000/2123/1/LIBRO-DIP-039.pdf · Quedan todos los derechos reservados. Esta publicación no puede ser reproducida, distribuida,

3.7 MULTIPLICACIÓN DE NÚMEROS COMPLEJOS ....................................................................................................... 66 3.8 DIVISIÓN DE NÚMEROS COMPLEJOS .................................................................................................................. 66 3.9 POTENCIAS Y RAÍCES DE NÚMEROS COMPLEJOS .................................................................................................. 67

3.9.1 Extracción de la raíz cuadrada de un número complejo .................................................................... 67 3.10 OPERACIONES DE LOS NÚMEROS COMPLEJOS DADOS EN FORMA TRIGONOMÉTRICA .................................................... 68 3.11 FORMA EXPONENCIAL DE UN NÚMERO COMPLEJO ............................................................................................... 69 3.12 APLICACIONES .............................................................................................................................................. 69 3.13 SUBSECCIÓN. - EVALUACIÓN ........................................................................................................................... 70 3.14 EJERCICIOS PARA CALCULAR ............................................................................................................................ 71 3.15 RESPUESTA DE LOS EJERCICIOS PROPUESTOS EN LAS ACTIVIDADES DE APRENDIZAJE ..................................................... 72

CAPITULO IV. MATRICES, SISTEMAS LINEALES Y NO LINEALES ........................................................................... 73

4.1 INTRODUCCIÓN ....................................................................................................................................... 73 4.2 CONCEPTO DE MATRIZ ................................................................................................................................... 73

4.2.1. Tipos de matrices ................................................................................................................................ 74 4.2.2. Propiedades de la suma de matrices .................................................................................................. 75 4.2.3. Productos entre matrices .................................................................................................................... 76

4.3 DEFINICIÓN DE DETERMINANTE ....................................................................................................................... 77 4.4 REGLA DE SARRUS ......................................................................................................................................... 77 4.5 PROPIEDADES DE LOS DETERMINANTES .............................................................................................................. 78 4.6 MATRIZ INVERSA .......................................................................................................................................... 79 4.7 PROBLEMAS DE APLICACIÓN ............................................................................................................................ 80

4.7.1. Sistemas de Ecuaciones Lineales ........................................................................................................ 81 4.7.2. Resolución de sistema de ecuaciones lineales .................................................................................... 82

4.8 SISTEMAS DE ECUACIONES NO LINEALES ............................................................................................................. 84

4.8.1 Clasificación de los sistemas de inecuaciones de dos variables…………………………………………………82

4.9 DEFINICIÓN DE INTERVALOS ............................................................................................................................ 85 4.10 RESOLUCIÓN DE INECUACIONES ....................................................................................................................... 87 4.11 VALOR ABSOLUTO ........................................................................................................................................ 88 4.12 ECUACIONES CUADRÁTICAS ............................................................................................................................. 89

4.12.1 Gráfica de una ecuación cuadrática……………………………………………………………………………………………87

Ejemplo 2: Función Cuadrática con una Raíz Real ............................................................................................ 91 4.13 SUMA Y PRODUCTO DE LAS RAÍCES.................................................................................................................... 92 4.14 PROBLEMAS DE APLICACIÓN ............................................................................................................................ 95 4.15 SISTEMAS DE ECUACIONES CUADRÁTICOS .......................................................................................................... 96 4.16 OPERACIONES CON MATRICES ......................................................................................................................... 97 4.17 APLICACIONES .............................................................................................................................................. 98 4.18 APLICACIONES DEL ALGORITMO DE SOLUCIÓN DE ECUACIONES LINEALES A LA RESOLUCIÓN DE PROBLEMAS ..................... 99 SOLUCIÓN DE ECUACIONES LINEALES A LA RESOLUCIÓN DE PROBLEMAS ................................................................................. 99 4.19 APLICACIONES UTILIZANDO EL MÉTODO DE GAUSS ............................................................................................. 100 4.20 APLICACIONES DE LAS MATRICES .................................................................................................................... 101 4.21 EJERCICIOS PARA REPASO ............................................................................................................................. 103 4.22 RESOLUCIÓN DE PROBLEMAS QUE CONDUCEN A ECUACIONES LINEALES ................................................................. 106 4.23 RESOLUCIÓN DE PROBLEMAS Y EJERCICIOS QUE CONDUCEN A ECUACIONES CUADRÁTICAS ......................................... 107 4.24 EVALUACIÓN .............................................................................................................................................. 108 4.25 EJERCICIOS Y PROBLEMAS SOBRE ECUACIONES CUADRÁTICAS .............................................................................. 112

REFERENCIAS BIBLIOGRÁFICAS ........................................................................................................................ 116

Page 9: repositorio.unesum.edu.ecrepositorio.unesum.edu.ec/bitstream/53000/2123/1/LIBRO-DIP-039.pdf · Quedan todos los derechos reservados. Esta publicación no puede ser reproducida, distribuida,

Índice de Tablas

Tabla 1: Orden de los operadores lógicos. .............................................................................................. 15

Tabla 2: Función Lógica OR. ..................................................................................................................... 20

Tabla 3: Función Lógica AND. .................................................................................................................. 21

Tabla 4: Complemento de los elementos. .............................................................................................. 21

Tabla 5: Funciones Booleanas. ................................................................................................................ 22

Tabla 6: Mini términos. ........................................................................................................................... 24

Tabla 7: Tabla de verdad. ........................................................................................................................ 24

Tabla 8: (a+b+c)=a ̅+ b*c. ........................................................................................................................ 28

Tabla 9: f(x,y,z)=x+y* z .̅ ........................................................................................................................... 29

Tabla 10: f(x,y,z)=x+y ̅ *z. ......................................................................................................................... 29

Tabla 11: Ejemplos de aplicaciones. ........................................................................................................ 30

Índice de Figuras

Figura 1: Funciones Booleanas. ...................................................................................................... 23

Figura 2: Circuito Lógico. ................................................................................................................ 23

Figura 3: Maxi términos. ................................................................................................................ 25

Figura 4: Funciones booleanas. ...................................................................................................... 27

Figura 5: Represente gráficamente los siguientes postulados.......................................................... 28

Figura 6: Gráfica de los circuitos OR. .............................................................................................. 31

Figura 7: Gráfica de los circuitos AND. ............................................................................................ 32

Figura 8: Conjuntos no vacíos. ....................................................................................................... 38

Figura 9: gráfica 3.1. ...................................................................................................................... 86

Figura 10: grafica 3.2. .................................................................................................................... 86

Figura 11: grafica 3.3. .................................................................................................................... 86

Figura 12: grafica 3.4. .................................................................................................................... 86

Figura 13: grafica 3.5. .................................................................................................................... 87

Page 10: repositorio.unesum.edu.ecrepositorio.unesum.edu.ec/bitstream/53000/2123/1/LIBRO-DIP-039.pdf · Quedan todos los derechos reservados. Esta publicación no puede ser reproducida, distribuida,
Page 11: repositorio.unesum.edu.ecrepositorio.unesum.edu.ec/bitstream/53000/2123/1/LIBRO-DIP-039.pdf · Quedan todos los derechos reservados. Esta publicación no puede ser reproducida, distribuida,

11

PRÓLOGO

La Matemática es de las ciencias más antiguas, nacidas en la aurora de la civilización humana bajo la influencia de las crecientes necesidades prácticas, sociales, científicas y tecnológicas. El estudio lógico-histórico del desarrollo de la Matemática, define un armonioso sistema lógico-abstracto capaz de integrarse al complejo sistema de conocimientos científico-tecnológicos definido por otras ciencias (naturales, técnicas, sociales), que al emplear las teorías y los métodos de la Matemática, le plantean a ella nuevos problemas que estimulan su estudio, y cuyas soluciones contribuyen a su autodesarrollo.

La educación en nuestros días dirigida al logro de competencias, habilidades y la formación de valores, enfrenta a la comunidad científica (matemáticos, psicólogos, pedagogos y educadores matemáticos, entre otros) a complejas interrogantes: ¿Para quién enseñamos Matemática? ¿Qué Matemática enseñar?, ¿Cómo enseñar Matemática? ¿Cómo aprender Matemática?

Al intentar dar respuesta a las interrogantes presentadas, aparece la dicotomía: contextualizar la Matemática sin que su carácter lógico-abstracto, de generalización y rigor se debilite.

A lo anterior se une la diversidad de los estudiantes que comienzan sus estudios universitarios, relativo a: procedencia social y características del nivel de la enseñanza precedente. Lo anterior define dos planos de dificultades: el de los estudiantes, porque no es posible garantizarles ciertos parámetros comunes para su formación; y el de los docentes, porque dificulta el intercambio y la comunicación de experiencias pedagógicas.

La obra que se presenta tiene entre sus objetivos unificar el nivel en Matemática de los estudiantes que comienzan sus estudios universitarios, esperamos que el texto sirva de ayuda complementaria a todos aquellos estudiantes que se enfrentan a la resolución de problemas que requieran el uso de temas de nivel básico en Matemática, los que en dependencia de la especialidad se enfrentarán al estudio de disciplinas de Matemática Superior y de Matemática aplicada.

En la elaboración de este libro “Elementos de Matemática Básica para Carreras Universitarias, tomo II”, se tuvieron en cuenta, entre otros, los aspectos siguientes:

La contextualización de los contenidos matemáticos en la práctica, al considerar los principios didácticos del proceso enseñanza-aprendizaje y las relaciones interdisciplinarias con materias a las cuales la Matemática sirve de base.

Se establece un lenguaje claro, preciso, cercano y ameno que, sin perder el rigor científico, le permite al estudiante apropiarse de la base teórico-conceptual necesaria a través de ilustraciones y ejemplos demostrativos para enfrentarse a la resolución de variados ejercicios propuestos que garantizan la ejercitación y sistematización de los contenidos.

La obra también puede ser usada en temas seleccionados de profundización para los cursos del bachillerato.

Esperamos que esta segunda edición complemente la primera y contribuya a mejorar las experiencias del aprendizaje sistemático de las matemáticas a un nivel básico. Agradeceremos todos los aportes que puedan hacernos para, a su vez, mejorar este instrumento didáctico, así como las próximas ediciones.

LOS AUTORES

Page 12: repositorio.unesum.edu.ecrepositorio.unesum.edu.ec/bitstream/53000/2123/1/LIBRO-DIP-039.pdf · Quedan todos los derechos reservados. Esta publicación no puede ser reproducida, distribuida,

12

Page 13: repositorio.unesum.edu.ecrepositorio.unesum.edu.ec/bitstream/53000/2123/1/LIBRO-DIP-039.pdf · Quedan todos los derechos reservados. Esta publicación no puede ser reproducida, distribuida,

13

CAPÍTULO I. LÓGICA MATEMÁTICA

1.1. INTRODUCCIÓN

Este módulo aporta los conocimientos matemáticos para entender, inferir, aplicar y desarrollar modelos matemáticos tendientes a resolver problemas en el área de las ciencias. El estudiante resolverá problemas utilizando las técnicas básicas de lógica proposicional, recurriendo a sus leyes y diagramas.

Las unidades temáticas que se despliegan en este módulo, tienen carácter teórico-práctico, pues contribuyen a lograr una interrelación apropiada entre la dirección racional y emocional del comportamiento de los estudiantes, desarrollan rasgos del carácter y hábitos del pensar, estimulan la movilidad de los procesos del pensamiento, favorecen la coherencia y precisión al expresar una idea del lenguaje común al matemático y viceversa y capacitan para la valoración crítica del trabajo, tanto propio como de sus condiscípulos.

Es importante, desde la definición de las proposiciones lógicas, contextualizar el contenido con los procesos vitales-escolares y propiciar la lectura de conceptos y relaciones de la Lógica Proposicional con las demás ciencias. A partir de las proposiciones y su valor de verdad, se desarrollan las operaciones lógicas básicas tales como: la negación, la conjunción, la disyunción, el condicional, el bicondicional, para concluir si se trata de tautologías, contradicciones o contingencias.

Se introduce el Álgebra Binaria de Boole, con el objetivo de analizar de manera simple y en forma alfanumérica las actividades u operaciones diarias; donde las actividades o elementos que intervienen en un proceso (y que tienen cada uno de ellos dos alternativas, como estar energizados o no, estar en óptimas condiciones o no, entre otros), son reemplazados en una ecuación y son representados por símbolos numéricos o alfanuméricos que permiten la construcción de circuitos lógicos.

1.1.1 OBJETIVOS

Resolver problemas utilizando las técnicas básicas de la lógica matemática.

Desarrollar el razonamiento lógico para emplearlo en la demostración de teoremas de las ciencias.

Resolver circuitos lógicos en serie y paralelo por medio de Lógica Matemática.

1.1.2 Conceptos Fundamentales

Dentro del módulo se abordarán los siguientes conceptos y definiciones:

Lógica Matemática:

1.1. Proposiciones

1.2. Operadores lógicos

1.3. Orden de los operadores lógicos

1.4. Tablas de verdad

1.5. Tautología, contradicción y contingencia

1.6. Equivalencia e implicación lógica

1.7. Leyes del álgebra de las proposiciones y aplicaciones.

1.8. Funciones y polinomios booleanos.

1.9. Funciones canónicas mintérminos y maxtérminos y sus circuitos lógicos correspondientes en series. y en paralelo.

Page 14: repositorio.unesum.edu.ecrepositorio.unesum.edu.ec/bitstream/53000/2123/1/LIBRO-DIP-039.pdf · Quedan todos los derechos reservados. Esta publicación no puede ser reproducida, distribuida,

14

1.1.3 Desarrollo de los conceptos fundamentales

Lógica Matemática:

Lógica (concepto). La Lógica es la Ciencia que expone las leyes, modos y formas de raciocinio.

Aporte de la Lógica a la Matemática

De acuerdo al concepto anterior, podemos asegurar que la simbología que usa la lógica, ayuda a la Matemática en todos sus razonamientos.

¿Qué es una proposición?

Una proposición es toda oración de la cual se puede decir que es verdadera o falsa, es decir, es un enunciado coherente que posee un valor de verdad: verdadero (v) o falso (f), sin ambigüedades y en determinado contexto.

Ejemplo:

p. Hoy es lunes (en dependencia del día en cuestión es verdadero o falsa)

(2+3) ² = 4 + 9 (falso)

Lima es una ciudad de la costa del Perú. (Verdadero)

*Se simboliza con letras minúsculas (p; q; r; u otras del alfabeto)

1.1.4 Ejemplificando

Identifica las expresiones que son proposiciones:

Vladimir Putin es presidente de Rusia.

Tal vez compre un obsequio.

Formuló una pregunta difícil de responder.

3 + 2 = 5.

Dos números enteros distintos pueden sumar cero.

¡Ojalá tomen lo que he estudiado!

1.1.5 Respuestas

Son proposiciones: 1; 3; 4 y 5. Las demás no son expresiones coherentes, tienen ambigüedades.

1.2. Tipos de proposiciones

Simples: son aquellas que tienen una única idea, es decir una sola afirmación, siempre en positivo.

Ejemplo:

-6 es un número entero.

Los universitarios tienen carnet de medio pasaje.

Compuestas: son aquellas que tienen dos o más proposiciones.

Ejemplo: Quito está en Ecuador y Ecuador está en Sudamérica

Si x² =81 → x= - 9 o x= 9

Page 15: repositorio.unesum.edu.ecrepositorio.unesum.edu.ec/bitstream/53000/2123/1/LIBRO-DIP-039.pdf · Quedan todos los derechos reservados. Esta publicación no puede ser reproducida, distribuida,

15

Ejercitándonos:

Identifica si la proposición es compuesta (C) o simple (S).

Rosa es abogada.

Dos no es mayor que 6.

Los cuadriláteros tienen cuatro lados.

Alberto tiene 20 años.

Juan y José están en Chile.

No es cierto que 234 sea igual a 243.

Respuestas: son simples: 1 y 4; son compuestas: 2; 3; 5 y 6.

1.3. Operadores lógicos: (llamados también conectores lógicos) son palabras que sirven para enlazar proposiciones simples o cambiar el valor de verdad de una proposición

Tabla 1: Orden de los operadores lógicos.

CONECTOR/OPERADOR

SÍMBOLO ESQUEMA SIGNIFICADO VALOR DE VERDAD

CONJUNCIÓN pq p y q Verdadera (V) si ambas

proposiciones son V

DISYUNCIÓN INCLUSIVA

pq p o q Falsas (F) solo si ambas

proposiciones son F

DISYUNCIÓN EXCLUSIVA

p q o p o q Falsas (F) si ambas

proposiciones

tienen igual valor de verdad

CONDICIONAL pq si p, entonces q Falsas (F) solo si la primera

proposición es V y la segunda es F

Page 16: repositorio.unesum.edu.ecrepositorio.unesum.edu.ec/bitstream/53000/2123/1/LIBRO-DIP-039.pdf · Quedan todos los derechos reservados. Esta publicación no puede ser reproducida, distribuida,

16

NEGACIÓN p no p Lo opuesto al valor de la

Proposición

Ejemplificando

Dadas las siguientes proposiciones:

p: Estudio sistemáticamente

q: Obtendré buenas calificaciones en Álgebra

r: Voy a bailar todos los fines de semana

s: Me sentiré feliz

Escriba con palabras la siguiente proposición: (~ p r) ~ q

Respuestas: si no estudio sistemáticamente y voy a bailar todos los fines de semana entonces no obtendré buenas calificaciones en Álgebra.

2. Dadas las siguientes proposiciones:

p: a es un número par

q: 2a es un número par

r: a es un múltiplo de 6

s: a < 10

Escribe con símbolos la siguiente proposición:

Si a es un número par y múltiplo de 6, entonces 2a es par o a es menor que 10

Respuesta:

1.4. Tablas de valores de verdad

Una tabla de valores de verdad de una proposición, es una tabla que se arma con los posibles valores de verdad de las proposiciones simples que la componen, con la finalidad de obtener el valor de verdad de la proposición dada.

Cantidad de valores de verdad que debe llevar una tabla:

O sea que, si el número de proposiciones simples que componen una proposición es 5, los valores de verdad serán:

Ejemplos de tablas de valores de verdad:

La negación

La negación de la proposición p es ~p, cuya tabla de valores de verdad es la siguiente:

sproposionen

nesproposicionAvaloresn

º2

º2'º

3225 valoresnº

Page 17: repositorio.unesum.edu.ecrepositorio.unesum.edu.ec/bitstream/53000/2123/1/LIBRO-DIP-039.pdf · Quedan todos los derechos reservados. Esta publicación no puede ser reproducida, distribuida,

17

P ~ p

V

F

F V

Como conclusión podemos decir que la negación es verdadera si la proposición simple es falsa y viceversa.

La disyunción o suma lógica

La disyunción de las proposiciones p y q es la proposición pvq, donde p y q se llaman disyuntivos, cuya tabla de valores de verdad es la siguiente:

p: estudio

q: veo TV

p v q: estudio o veo TV

Como conclusión podemos decir que la disyunción es verdadera si al menos uno de los disyuntivos también lo es.

El condicional o la implicación

El condicional de las proposiciones p y q es la proposición pq, donde p se llama antecedente y q consecuente, cuya tabla de valores de verdad es la siguiente:

p: apruebo

q: te presto el libro

p q: apruebo, entonces te presto el libro

Como conclusión podemos decir que el condicional es falso si el antecedente es verdadero y el consecuente es falso (2º línea de la tabla).

Condiciones necesarias y suficientes

p condición SUFICIENTE para q (q si p)

q condición NECESARIA para p (p solo si q)

El bicondicional o la doble implicación

El bicondicional de las proposiciones p y q es la proposición pq, cuya tabla de valores de verdad es la siguiente:

p Q p q

V V V

V F V

F V V

F F F

p Q p q

V V V

V F F

F V V

F F V P q p q

V

V

F

F

V

F

V

F

V

F

V

V

p Q p q

Page 18: repositorio.unesum.edu.ecrepositorio.unesum.edu.ec/bitstream/53000/2123/1/LIBRO-DIP-039.pdf · Quedan todos los derechos reservados. Esta publicación no puede ser reproducida, distribuida,

18

p: apruebo

q: te presto el libro

p q: solamente si apruebo, te presto el libro

Como conclusión podemos decir que el bicondicional es verdadero si los valores de verdad de las proposiciones simples que la componen son iguales.

La diferencia simétrica

La diferencia simétrica de las proposiciones p y q es la proposición p ∆ q, cuya tabla de valores de verdad es la siguiente:

p: estudio

q: veo TV

p ∆ q: estudio o bien veo TV

Como conclusión podemos decir que la diferencia simétrica es verdadera si los valores de verdad de las proposiciones simples que la componen son distintos.

Tautología

Definición: Se dice que una proposición es una tautología, si es verdadera independientemente de los valores de verdad de las proposiciones simples que la componen: ↓

p Q (pq) [(pq) (q p)]

V

V

F

F

V

F

V

F

V

F

F

V

V

V

V

V

V

F

V

V

V

F

F

V

V

V

F

V

Contradicción

Definición. Una proposición es una contradicción, si es falsa independientemente de los valores de verdad de las proposiciones simples que la componen.

Por ejemplo: ↓

V

V

F

F

V

F

V

F

V

F

F

V

q P p ∆ q

V

V

F

F

V

F

V

F

F

V

V

F

Page 19: repositorio.unesum.edu.ecrepositorio.unesum.edu.ec/bitstream/53000/2123/1/LIBRO-DIP-039.pdf · Quedan todos los derechos reservados. Esta publicación no puede ser reproducida, distribuida,

19

p Q (p q) - [(p q) (q p)]

V

V

F

F

V

F

V

F

V

F

F

V

F

F

F

F

F

V

V

F

V

F

V

V

V

F

F

V

V

V

F

V

Contingencia

Definición. Una proposición es una contingencia si no es ni verdadera ni falsa independientemente de los valores de verdad de las proposiciones simples que la componen.

P Q (p q) v [(p q) (q p)]

V

V

F

F

V

F

V

F

V

F

F

V

V

F

F

V

V

F

V

V

V

F

F

V

V

V

F

V

Leyes lógicas. Una ley lógica es una proposición verdadera.

1º) Involución. La negación de la negación de una proposición, es equivalente a la misma proposición.

P -(-p) p

V

F

V

F

F

V

V

V

2º) Idempotencia de la conjunción. La conjunción de una misma proposición es equivalente a la misma proposición.

P (p p) p

V

F

V

F

V

V

Page 20: repositorio.unesum.edu.ecrepositorio.unesum.edu.ec/bitstream/53000/2123/1/LIBRO-DIP-039.pdf · Quedan todos los derechos reservados. Esta publicación no puede ser reproducida, distribuida,

20

3a) La doble implicación y la implicación. La doble implicación es equivalente a la conjunción de la implicación y su recíproca.

p Q (p q) [(pq) (qp)

V

V

F

F

V

F

V

F

V

F

F

V

V

V

V

V

V

F

V

V

V

F

F

V

V

V

F

V

4a) La diferencia simétrica y la doble implicación. La diferencia simétrica es equivalente a la negación de la doble implicación.

p Q (p q) - (p q)

V

V

F

F

V

F

V

F

F

V

V

F

V

V

V

V

F

V

V

F

V

F

F

V

1.5. Funciones y polinomios booleanos.

En 1815 George Boole propuso una herramienta matemática llamada Álgebra de Boole. Luego en 1938 Claude Shannon propuso que con esta álgebra es posible modelar los llamados Sistemas Digitales. El Álgebra de Boole es un sistema matemático que utiliza variables y operadores lógicos. Las variables pueden valer 0 o 1. Y las operaciones básicas son OR (+) y AND (·). Luego se definen las expresiones de conmutación como un finito de variables y constantes, relacionadas mediante los operadores (AND y OR). En la ausencia de paréntesis, se utilizan las mismas reglas de precedencia, que tienen los operadores suma (OR) y multiplicación (AND) en el álgebra normal.

Una Álgebra de Boole Bivalente se define sobre un conjunto de dos elementos B={0,1}, con reglas para los operadores binarios + y. de la siguiente manera:

Tabla 2: Función Lógica OR.

X Y x + y

0 0 0

Page 21: repositorio.unesum.edu.ecrepositorio.unesum.edu.ec/bitstream/53000/2123/1/LIBRO-DIP-039.pdf · Quedan todos los derechos reservados. Esta publicación no puede ser reproducida, distribuida,

21

0 1 1

1 0 1

1 1 1

Tabla 3: Función Lógica AND.

X Y x . y

0 0 0

0 1 0

1 0 0

1 1 1

Tabla 4: Complemento de los elementos.

X �̅�

0 1

1 0

Leyes del Álgebra de Boole.

1) Conmutatividad:

X + Y = Y + X

X · Y = Y · X

2) Asociatividad:

X + (Y + Z ) = (X + Y ) + Z

X · (Y · Z ) = (X · Y ) · Z

3) Distributividad:

X + (Y · Z ) = (X + Y ) · (X + Z )

X · (Y + Z ) = (X · Y ) + (X · Z )

4) Elementos Neutros (Identidad):

X + 0 = X

X · 1 = X

5) Complemento:

Page 22: repositorio.unesum.edu.ecrepositorio.unesum.edu.ec/bitstream/53000/2123/1/LIBRO-DIP-039.pdf · Quedan todos los derechos reservados. Esta publicación no puede ser reproducida, distribuida,

22

x 1

0 0 0 0

0 0 1 0

0 1 0 0

0 1 1 0

1 0 0 0

1 0 1 0

1 1 0 0

1 1 1 0

X + x̅ = 1

X · x̅ = 0

6) Dominación:

X + 1 = 1 X · 0 = 0

7) Idempotencia:

X + X = X X · X = X

1.6. Funciones Booleanas

Una variable binaria puede tomar el valor 0 o 1. Una función de Boole es una función formada con variables binarias, dos operadores binarios OR y AND, el operador NOT, el paréntesis y el signo igual. Para un valor dado de variables, la función puede ser 0 o 1. considérese por ejemplo la función de Boole:

F1 = xy z̅

La función F1 es igual a 1 si x=1; y=1 y z̅=1; de otra manera F1=0

Una función de Boole puede ser representada por medio de una tabla de verdad.

Tabla de verdad para: F1 = xy z̅

Tabla 5: Funciones Booleanas.

Una función de Boole puede ser transformada de una expresión algebraica a un diagrama lógico compuesto de compuertas AND, OR y NOT.

Diagrama lógico para: F1 = xy z̅

Page 23: repositorio.unesum.edu.ecrepositorio.unesum.edu.ec/bitstream/53000/2123/1/LIBRO-DIP-039.pdf · Quedan todos los derechos reservados. Esta publicación no puede ser reproducida, distribuida,

23

Figura 1.Funciones Booleanas.

Ejemplos: Simplifique la siguiente función de Boole al mínimo número de literales:

x + x̅ y

x (x̅ + y)

RESPUESTAS:

x + x̅ y = (x +x̅ )(x + y) = 1 . (x + y) = x + y

x (x̅ + y) = xx̅ + x y = 0 + x y = x y

1.7. Funciones canónicas mintérminos y maxtérminos y sus circuitos lógicos correspondientes en series y en paralelo.

Correspondencia de la lógica combinacional.

Una función lógica presenta una correspondencia “uno a uno” con un circuito lógico o con una tabla de verdad.

Ejemplo: Sea la siguiente función lógica:

El circuito lógico y su tabla de verdad serán:

Figura 2. Circuito Lógico.

dcacbaz ).().(

Page 24: repositorio.unesum.edu.ecrepositorio.unesum.edu.ec/bitstream/53000/2123/1/LIBRO-DIP-039.pdf · Quedan todos los derechos reservados. Esta publicación no puede ser reproducida, distribuida,

24

1.8. Mini términos

Una función combinacional distintiva son los minitérminos de “n” variables, y se los denota como mí. Son funciones booleanas cuya tabla de verdad tiene un “1” en la i-ésima fila, y un “0” en las restantes.

Tabla 6: Mini términos.

1.8.1 Forma canónica “suma de mini términos”

Dada una función z de “n” variables, cuya tabla de verdad tiene “1” en las filas a, b, ..., k, y “0” en las demás. A partir de la definición de mini término, y usando la función OR, es evidente que:

EJEMPLO:

Sean las funciones para z1=Z1(A, B, C, D), z2=Z2(A, B, C, D) y z3=Z3(A, B, C, D), caracterizadas por la siguiente tabla de verdad, determinar las funciones booleanas correspondientes:

Tabla 7: Tabla de verdad.

Solución: Aplicando el concepto de minitérminos, las funciones buscadas serán:

Page 25: repositorio.unesum.edu.ecrepositorio.unesum.edu.ec/bitstream/53000/2123/1/LIBRO-DIP-039.pdf · Quedan todos los derechos reservados. Esta publicación no puede ser reproducida, distribuida,

25

EJEMPLO DE MINITÉRMINOS

1001 a b̅ c̅ d = 1*0̅ *0̅ * 1= 1

1100 a b c̅ d̅ = 1*1*0̅ * 0̅ = 1

0010 a̅ b̅ c d̅ = 0*0*1 * 0̅ = 1

1111 a b c d = 1*1*1*1 = 1

0000 a̅ b̅ c̅ d̅ = 0̅ 0̅ 0̅ 0̅ =1

1.9. Construcción algebraica

Cualquier expresión booleana puede convertirse a su forma canónica “suma de mini términos” empleando las propiedades del Álgebra de Boole. A esta forma canónica también suele denominarse “Suma De Productos (SDP)”.

Ejemplo: Encontrar la forma canónica “suma de mini términos” de:

Solución:

ddcbaddcbaaddcbbaz ,

O bien:

1.9.1 Maxi términos

Una segunda función son los maxi términos de “n” variables, denotada como Mi. Son funciones booleanas cuya tabla de verdad tiene un “0” en la i-ésima fila, y un “1” en las restantes.

.

Figura 3: Maxi términos.

dcbadcbadcbadcbadcbadcbadcbadcbaz

cbacbcaz

Page 26: repositorio.unesum.edu.ecrepositorio.unesum.edu.ec/bitstream/53000/2123/1/LIBRO-DIP-039.pdf · Quedan todos los derechos reservados. Esta publicación no puede ser reproducida, distribuida,

26

1.10. Forma canónica “producto de maxitérminos”

Toda función z tiene un conjunto único de maxi términos Mí, que corresponde al conjunto de ceros que aparecen en la columna de salida de su tabla de verdad. La forma canónica de producto de maxi términos será la función AND o producto lógico de estos maxi términos. A esta forma canónica también suele denominarse “Producto De Sumas (PDS)”.

Ejemplo: Sea la la siguiente función booleana de tres variables:

cbaz

La expresión canónica de producto de maxi términos será:

Ejemplo maxi términos

1001 a̅ + b + c + d̅ = 1̅ + 0 + 0 + 1̅ = 0

1100 a̅ + b̅ + c + d = 1̅ + 1̅ + 0 + 0 = 0

0010 a + b + c̅ + d = 0 + 0 + 1̅ + 0 = 0

0000 a + b + c + d = 0 + 0 + 0 + 0 = 0

1111 a̅ + b̅ + c̅ + d̅ = 1̅ + 1̅ + 1̅ + 1̅ = 0

Circuitos combinacionales

Las formas canónicas anteriores se representan con circuitos combinacionales de dos niveles de compuertas:

Notación decimal

Las funciones booleanas, dadas en cualesquiera de sus formas canónicas, pueden escribirse de manera

simplificada usando el símbolo para indicar la suma de productos, y para el producto de sumas.

Page 27: repositorio.unesum.edu.ecrepositorio.unesum.edu.ec/bitstream/53000/2123/1/LIBRO-DIP-039.pdf · Quedan todos los derechos reservados. Esta publicación no puede ser reproducida, distribuida,

27

Figura 4: Funciones booleanas.

EJERCICIO 1: Represente gráficamente los siguientes postulados booleanos:

A+0 = A

A+1=1

A*0=0

A*1=A

A+A=A

SOLUCIÓN:

a.

b.

c.

d.

e.

A=1

0 A=0

A=0

X=1 X=0

A=1

1

A=0

A=1 X=1 X=0

A=1

0

A=0

0 x=0 x=0

A=0

1

A=1

1

x=0 x=1

A=1

1

A=0

A=0 X=1 X=0

Page 28: repositorio.unesum.edu.ecrepositorio.unesum.edu.ec/bitstream/53000/2123/1/LIBRO-DIP-039.pdf · Quedan todos los derechos reservados. Esta publicación no puede ser reproducida, distribuida,

28

EJERCICIO 2: Deduzca los postulados a partir del gráfico:

Solución: A*1=A

Solución: A+A=A

Figura 5: Represente gráficamente de postulados booleanos

EJERCICIO 3. Construya la tabla de verdad de:

1) (a+b+c)= a̅ + b*c 2) f(x,y,z)=x+y* z̅ 3) f(x,y,z)=x+y̅ *z

1) Solución:

Tabla 8: (a+b+c)=a ̅ + b*c.

A B C �̅� F

0 0 0 1 1

0 0 1 1 1

0 1 0 1 1

0 1 1 1 1

1 0 0 0 0

1 0 1 0 0

1 1 0 0 0

1 1 1 0 1

A=1

1

A=1

1

x=1 x=1

A=1

1

A=0

A=0 X=1 X=0

1

Page 29: repositorio.unesum.edu.ecrepositorio.unesum.edu.ec/bitstream/53000/2123/1/LIBRO-DIP-039.pdf · Quedan todos los derechos reservados. Esta publicación no puede ser reproducida, distribuida,

29

2) Solución

Tabla 9: f(x,y,z)=x+y* z ̅.

X Y Z �̅� F

0 0 0 1 0

0 0 1 0 0

0 1 0 1 1

0 1 1 0 0

1 0 0 1 1

1 0 1 0 1

1 1 0 1 1

1 1 1 0 1

3) Solución

Tabla 10: f(x,y,z)=x+y ̅ *z.

X Y Z �̅� F

0 0 0 1 0

0 0 1 1 1

0 1 0 0 0

0 1 1 0 0

1 0 0 1 1

1 0 1 1 1

1 1 0 0 1

1 1 1 0 1

1.11. Aplicaciones

La Lógica Matemática es una ciencia formal que estudia las propiedades y relaciones entre entidades abstractas con objetos, números y símbolos. En la sociedad moderna se necesita transmitir información de forma segura. Aquí la lógica matemática, las teorías de conjuntos, de códigos y la

Page 30: repositorio.unesum.edu.ecrepositorio.unesum.edu.ec/bitstream/53000/2123/1/LIBRO-DIP-039.pdf · Quedan todos los derechos reservados. Esta publicación no puede ser reproducida, distribuida,

30

criptología son herramientas imprescindibles. Sin ellas, no sería posible transmitir, por ejemplo, imágenes desde los satélites.

En medicina, se puede aplicar la propiedad reflexiva condicional y bicondicional para el tratamiento de cálculos renales. Por otro lado, modelos matemáticos ayudan a estudiar las redes neuronales, facilitando la comprensión de los mecanismos cerebrales del aprendizaje. En Arquitectura, con el empleo de los grupos cristalográficos, a través de las operaciones lógicas podemos generar figuras ornamentales distintas como colecciones de baldosas a partir de un mismo motivo ornamental. Al final vemos que aprender acerca de las matemáticas es muy fácil solo se necesita dedicación y aprender a manejarlas en la vida cotidiana, sobre la base de su significado social y su utilidad práctica.

Como resultado de aprendizaje de este módulo, el estudiante será capaz de construir un proceso lógico matemático para distinguir la verdad y la falsedad de las proposiciones; construye conjuntos y relacionarlos entre sí; demuestra conocimientos de la Lógica Proposicional; resuelve ejercicios integradores de aplicación de la Lógica Proposicional y la Teoría de Conjuntos; construyen circuitos lógicos en serie y paralelo por medio de Lógica Matemática; resuelven problemas tanto científicos como analíticos y demuestran dominio de las ciencias exactas.

Ejemplos de aplicaciones:

Aplicación de funciones canónicas en la construcción de circuitos lógicos:

EJERCICIO 1: Dada la siguiente tabla de valores de verdad determine las funciones canónicas min término y max término y represente el circuito lógico de cada uno de ellas.

Tabla 11: Ejemplos de aplicaciones.

Orden X Y Z F

0 0 0 0 0

1 0 0 1 0

2 0 1 0 1

3 0 1 1 1

4 1 0 0 0

5 1 0 1 0

6 1 1 0 1

7 1 1 1 1

RESPUESTAS:

MINTÉRMINO (1) MAXTÉRMINO (0)

m2 =x̅ y z̅ MO = x+ y+ z

m3 =x̅ y z M1 = x + y + z̅

m6 =x y z ̅ M4 = x̅ + y + z

m7 =x y z M5= x̅ + y + z̅

Page 31: repositorio.unesum.edu.ecrepositorio.unesum.edu.ec/bitstream/53000/2123/1/LIBRO-DIP-039.pdf · Quedan todos los derechos reservados. Esta publicación no puede ser reproducida, distribuida,

31

Y

Z

Y

X

Y

Z

X

Y

Z

Fm = (x̅ y z̅) +( x̅ y z)+( x y z ̅ ) + (x y z)

GRÁFICA DE LOS CIRCUITOS

Fm = (x̅ y z̅) +( x̅ y z)+( x y ) + (x y z) (SUMA DE PRODUCTOS)

Fm =∑ m2, m3, m6 , m7

X

Z

X

Figura 6: Gráfica de los circuitos OR.

PRODUCTO DE SUMAS

F xyz = (x + y + z) (x + y + z̅) (x̅ + y + z) (x̅ + y + z)̅

F xyz = M0 , M1, M4, M5

OR

Page 32: repositorio.unesum.edu.ecrepositorio.unesum.edu.ec/bitstream/53000/2123/1/LIBRO-DIP-039.pdf · Quedan todos los derechos reservados. Esta publicación no puede ser reproducida, distribuida,

32

Z

X

Y

Z

X

Y

Z

X

Y

X

Y

Z

Figura 7: Gráfica de los circuitos AND.

Ejercicios o problemas

1) Escriba verdadero (V) o falso (F) según corresponda en cada una de las siguientes aseveraciones:

a) La Lógica Matemática se caracteriza por emplear un lenguaje simbólico y realizar una abstracción de los contenidos. ( )

b) La Lógica Matemática se utiliza para demostrar teoremas. ( )

c)Una proposición se dice que es simple o atómica, si no está afectada por conectivos lógicos( )

d) En el sistema operacional lógico, el condicional no tiene el mismo significado que la implicación. ( )

1. a, F; b, V; c, V; d, F

2. a, V; b, V; c, V; d, V

3. a, F; b, F; c, V; d, F

4. a, V; b, V; c, V; d, F

RESPUESTA CORRECTA 4)

2) Sean las siguientes frases:

p: es relación de equivalencia

r: es reflexiva

s: es simétrica

t: es transitiva

La sentencia lógica p ↔ r ∧ s ∧ t, significa que:

a) Una relación es una relación de equivalencia si y sólo si no es reflexiva, simétrica y transitiva.

AND

Page 33: repositorio.unesum.edu.ecrepositorio.unesum.edu.ec/bitstream/53000/2123/1/LIBRO-DIP-039.pdf · Quedan todos los derechos reservados. Esta publicación no puede ser reproducida, distribuida,

33

b) Una relación es reflexiva, simétrica y transitiva.

c) Una relación es una relación de equivalencia si y sólo si es reflexiva, simétrica y transitiva.

d) Una equivalencia es reflexiva, simétrica y transitiva.

RESPUESTA CORRECTA c)

3) Dadas las siguientes frases:

p: necesita un doctor

q: necesita un abogado

r: tiene un accidente

s: está enfermo

u: está herido

La sentencia lógica (p ∧q) ↔ (s ∧ u), establece que:

a) Si está enfermo entonces necesita un doctor y si

tiene un accidente entonces necesita un abogado.

b) Necesita un doctor y un abogado, si y sólo sí, está enfermo y está herido.

c) Si no está enfermo ni está herido, entonces no necesita un doctor.

d) Si necesita un doctor y necesita un abogado entonces ha tenido un accidente.

RESPUESTA CORRECTA b)

4) Dado el siguiente esquema molecular:

Construir su tabla de valores de verdad y clasificarla según corresponda en:

a) Contradicción b) Tautológica c) Contingencia

5) Una traducción al lenguaje formal de “Jipijapa mejora su imagen si la Alcaldía realiza obras o los ciudadanos colaboran en el aseo de las calles”, siendo las proposiciones simples:

m: La Alcaldía realiza obras.

n: Los ciudadanos colaboran en el aseo de las calles.

p: Jipijapa mejora su imagen.

Es: → (𝒎 ∨ 𝒏)

a) Verdadero

b) Falso

RESPUESTA CORRECTA a)

p | q (p q) - [(p q) (q p)]

Page 34: repositorio.unesum.edu.ecrepositorio.unesum.edu.ec/bitstream/53000/2123/1/LIBRO-DIP-039.pdf · Quedan todos los derechos reservados. Esta publicación no puede ser reproducida, distribuida,

34

6) Simplifique la siguiente función de Boole al mínimo número de literales:

a). yxyzxzyx

b). x y +x̅ z + y z

RESPUESTAS:

a). yxyzxzyx = yxzxyxzxyxyyzx )1()(

b). x y +x̅ z + y z =

zxxyyzxzxy

yzxxyzzxxyxxyzzxxy

)1()1(

)(

7) Si E= {1,0}, razona cuáles de las afirmaciones siguientes son correctas y cuáles no:

a) {0} ∈E

b) ∅ ∈ E

c) {0} ⊂ E

d) 0 ∈ E

e) 0⊂ E.

8).Escribe simbólicamente las afirmaciones siguientes:

a) v pertenece al conjunto M

b) El conjunto T contiene como subconjunto al conjunto H

c) Entre los elementos del conjunto G no está el número 2

d) El conjunto Z no es un subconjunto del conjunto A

e) El conjunto X no contiene al conjunto K

f) El conjunto H es un subconjunto propio del conjunto K

RESPUESTA CORRECTA: a) {0} ∈ E, b) ∅∈ E, c) {0} ⊂ E, d) 0 ∈ E y e) 0 ⊂ E.

9) Si E= {1,0}, razona cuáles de las afirmaciones siguientes son correctas y cuáles no:

a) {0} ∈ E

b) ∅∈ E

c) {0} ⊂ E

d) 0 ∈ E

e) 0 ⊂ E.

RESPUESTA CORRECTA: Afirmaciones correctas: b); c); d); las demás son incorrectas.

RESPUESTA CORRECTA a)

10) Escriba verdadero (V) o falso (F) según corresponda en cada una de las siguientes aseveraciones:

Page 35: repositorio.unesum.edu.ecrepositorio.unesum.edu.ec/bitstream/53000/2123/1/LIBRO-DIP-039.pdf · Quedan todos los derechos reservados. Esta publicación no puede ser reproducida, distribuida,

35

I. Un conjunto definido por comprensión requiere escribir dentro de una llave una propiedad característica de los elementos del conjunto y solamente de ellos. ( )

II. La unión de una colección de conjuntos es el conjunto de todos los elementos contenidos al menos una vez en los conjuntos. ( )

III. Dados dos conjuntos A y B, la diferencia simétrica estará formada por todos los elementos no comunes a los conjuntos A y B. ( )

IV. La Lógica binaria trabaja con variables que toman más de dos valores discretos.

1.a, V; b, V; c, V; d, V

2.a, F; b, F; c, V; d, F

3.a, V; b, V; c, V; d, F

Respuesta correcta: 3)

11) Con el conjunto C= {x2 + 5x + 6 = 0}, los subconjuntos que se pueden formar son:

a) Ø, {2; 3} b) Ø; { -2; -3}; { -2}; { -3} c) Ø; {2; 3}; {2}; {3} d) { -2; -3}; { -2}; { -3}

Respuesta correcta: b)

12) Relacionado con las operaciones con conjuntos, enlace (vincule) los conceptos y las definiciones según corresponda para ciertos conjuntos A, B y el conjunto universal U:

I. Unión de conjuntos. 1. x ϵ A x ϵ B II. Intersección de conjuntos. 2. x ϵ A v x ϵ B

III. Complementos de conjuntos. 3. x ϵ A x B

IV. Diferencia de conjuntos. 4. x ϵ U x A

a) i,2; ii,1; iii,4; iv,3. b) i,4; ii,1; iii,2; iv,3. c) i,3; ii,2; iii,4; iv,3. d) i,2; ii,1; iii,3; iv,4.

Respuesta correcta a)

13) Dados los conjuntos:

M = {1; 3; 5; 7; 9} y L = {x/x es impar 1 ≤ x ≤ 9}, se puede afirmar que: (circule las respuestas correctas)

a) M ∩ L = M b) M ∩ L = ø c) M U L = M d) M U L = ø

Respuesta correcta a); c)

14) Considere los siguientes conjuntos:

A = {x/x ϵ N es par}, B = {y/y ϵ N es múltiplo de 2}, puede afirmarse: (seleccione la opción correcta)

Page 36: repositorio.unesum.edu.ecrepositorio.unesum.edu.ec/bitstream/53000/2123/1/LIBRO-DIP-039.pdf · Quedan todos los derechos reservados. Esta publicación no puede ser reproducida, distribuida,

36

a) A B; A = B; B A; B = A

b) A B; A ≠ B; B A; B ≠ A

c) A B; A ≠ B; B A; B ≠ A

Respuesta correcta a)

15) Sean los conjuntos: V={d}, W={c,d}, X={a,b,c}, Y={a,b} y Z={a,b,d}. Establece la veracidad de las siguientes afirmaciones, justificando en cada caso tu respuesta:

a)Y⊂X

b) W⊅V

c) W≠Z

d) Z⊃V

e) V⊄Y

f) Z⊅X

g) V⊂X

h) Y⊄Z

i) X=W

j) W⊂Y

Respuesta correcta: verdaderas (a;b;c;d;e;f). Falsas (g;h;i;j)

16) Sean los conjuntos A = {0, 1, 2, 3, 4, 5, 6, 7, 8, 9}; B = {-2, -1, 0, 1, 2, 3, 4}, C = {1, 3, 5, 7, 9} y D = {0, 2, 4, 6, 8}

16.1).- Diga si las siguientes afirmaciones son verdaderas (V) o falsas (F). Justifique las falsas.

a). -4 B b).5 C c).8 A d). 7 D

e). A B f).C A g).C B h). D A.

16.2).- Complete utilizando los símbolos ,, , , de forma tal que se obtenga una proposición verdadera.

a).3 ____ A b). -5 ____ B `c).6 ____ C d).8 ____ D

e). D ____ A f). B ____ A g).C ____ D h). A ____ C.

16.3)- Escriba en notación tabular el conjunto M = {x N \ -3,4 < x 8,286}.

Solución de los ejercicios propuestos del 16.1 al 16.3:

15.1. a) F. b) V c) V d) V e) F f) F g) F h) V

15.2. a) 3 A b) – 5 B c) 6 C d) 8 D e) D A f) B A g) C D

h) A C

15.3. M = {0, 1, 2, 3, 4, 5, 6, 7, 8}

Page 37: repositorio.unesum.edu.ecrepositorio.unesum.edu.ec/bitstream/53000/2123/1/LIBRO-DIP-039.pdf · Quedan todos los derechos reservados. Esta publicación no puede ser reproducida, distribuida,

37

ACTIVIDADES DE APRENDIZAJES

Ejercicios propuestos

1.- Si los valores de verdad de las proposiciones atómicas a, b y c, son respectivamente: 0, 0, 1. Indique el valor de verdad de las siguientes proposiciones moleculares.

Nota, asuma como 0 = Falso y 1 = Verdadero

a) (𝑎 Λ b)

b) ~(𝑎 → 𝑏)

c) (𝑎 𝛬 𝑏) → (𝑎 𝚅 𝑏)

RESPUESTAS:

a) (𝑎 Λ b): reemplazando los valores se tiene: (0 Λ 0) es 0. (F). Retroalimentación de la respuesta: la conjunción es verdadera si ambas proposiciones también lo son.

b) ~ (𝑎 → 𝑏): reemplazando los valores se tiene: ~ (0→0),): ~ (1) es 0. (F). Retroalimentación de la respuesta: el condicional es falso solo si el antecedente es verdadero y el consecuente es falso. La negación de la proposición 1 es ~1 que es 0.

c) (𝑎 𝛬 𝑏) → (𝑎 𝚅 𝑏): reemplazando los valores se tiene: (0 𝛬 0) → (0 𝚅 0) Retroalimentación de la respuesta: la conjunción es verdadera si ambas proposiciones también lo son, por tanto (0 𝛬 0) es F, luego la disyunción (0 𝚅 0) es F porque la disyunción es verdadera si al menos uno de los disyuntivos también lo es. Por tanto: F → F, es V (el condicional es falso si el antecedente es verdadero y el consecuente es falso)

2.- Indique si cada enunciado es o no una proposición:

¿Qué hora es?

¡Pare, por favor!

La edad de Gloria es 17 años.

Galápagos es considerado Patrimonio Cultural de la Humanidad.

Mi familia y yo viajaremos a la Sierra en fin de año.

Mi palabra se siente levantada por un caballo lírico que salta.

RESPUESTAS:

¿Qué hora es? No es proposición, es una pregunta.

¡Pare, por favor! No es proposición, es una orden.

La edad de Gloria es 17 años. Sí es proposición

Galápagos es considerado Patrimonio Cultural de la Humanidad. Sí es proposición

Mi familia y yo viajaremos a la Sierra en fin de año. Sí es proposición, pues puede ser V o F

Mi palabra se siente levantada por un caballo lírico que salta. No es proposición, es lenguaje lírico.

Recordar que una proposición es toda oración de la cual se puede decir que es verdadera o falsa, es decir, es un enunciado coherente que posee un valor de verdad: verdadero (v) o falso (f), sin ambigüedades.

Page 38: repositorio.unesum.edu.ecrepositorio.unesum.edu.ec/bitstream/53000/2123/1/LIBRO-DIP-039.pdf · Quedan todos los derechos reservados. Esta publicación no puede ser reproducida, distribuida,

38

A B

ǁǁǁ

C

3.- Dado el razonamiento (H1 ∧ H2) → C, donde:

H1: Si apruebo todas las materias entonces me voy de vacaciones por un mes.

H2: Me voy de vacaciones por un mes y compraré muchos recuerdos.

Una conclusión C que hace válido este razonamiento es:

No me voy de vacaciones por un mes.

Apruebo todas las materias y compraré muchos recuerdos.

No apruebo todas las materias y no me voy de vacaciones por un mes.

Me voy de vacaciones por un mes.

Apruebo todas las materias

RESPUESTAS: Apruebo todas las materias y compraré muchos recuerdos.

4.- Determine cuál de los siguientes conjuntos es vacío:

a) 𝐴 = {{∅}}

b) D= {∅}

c) 𝐵 = {∅, {∅}}

d) 𝐶 = {∅, ∅

e) 𝑀 = {𝑥⁄𝑥 ≠ 𝑥}

5.- Dados los siguientes conjuntos: U = {a, b, c, d, e, f, g, h, i}; A= {d, e, f, g}; B= {a, c, e, g, i}

Entonces (𝐴 − 𝐵) corresponde a:

a) (a, c, i)

b) (b, d, e, f, g, h)

c) (d, f)

d) (a, b, c, e, g, h, i)

e) Ninguna

6.- Sean A, B, C conjuntos no vacíos. Respecto del siguiente diagrama de Venn.

Figura 8: Conjuntos no vacíos.

La región sombreada corresponde a:

𝑎) (𝐴 ∩ 𝐵) − 𝐶; 𝑏) (𝐴 ∩ 𝐵) − 𝐴; 𝑐 ) (𝐴 ∪ 𝐵) − 𝐶; 𝑑) (𝐴 − 𝐵) ∩ 𝐶; 𝑒) (𝐵 − 𝐴) ∩ 𝐶

Page 39: repositorio.unesum.edu.ecrepositorio.unesum.edu.ec/bitstream/53000/2123/1/LIBRO-DIP-039.pdf · Quedan todos los derechos reservados. Esta publicación no puede ser reproducida, distribuida,

39

CAPÍTULO II GEOMETRÍA ANALÍTICA DEL PLANO

2.1 Introducción

El surgimiento de la Geometría Analítica la podemos encontrar en el trabajo de los antiguos griegos. Un antecedente parece encontrarse en las obras de Apolonio de Perga (269 – 190 a.n.e.), el cual en su libro “Las cónicas”, usa procedimientos muy similares a los que después llevaría a los matemáticos franceses René descartes (1596 – 1650) y a Pedro Fermat (1601 – 1665) a crear métodos que combinan armoniosamente los recursos del Álgebra y la Geometría Analítica.

Mediante esta idea Fermat llega al principio fundamental de la Geometría Analítica al expresar: “Siempre que en una ecuación final aparezcan dos variables, tenemos un lugar geométrico, pues el extremo de una de las variables describe una línea, recta o curva.”

Las cónicas poseen curiosas e interesantes propiedades por las que resultan sumamente útiles en la naturaleza, la ciencia, la técnica o el arte.

Por ejemplo, las órbitas de los planetas y cometas en su rotación alrededor del Sol son cónicas; los faros de los coches tienen sección parabólica, al igual que los hornos solares y las antenas de seguimiento de satélites, debido a que en la parábola los rayos que pasan por el foco salen paralelos al eje y viceversa. También existe un tipo de ayuda a la navegación (loran) basado en las propiedades de las hipérbolas.

Johannes Kepler (1571-1630), astrónomo y filósofo alemán, famoso por formular y verificar las tres leyes del movimiento planetario conocidas como leyes de Kepler. Johannes Kepler creía en la teoría heliocéntrica de Copérnico, según la cual la Tierra gira alrededor del Sol, que permanece estacionario. Kepler formuló una descripción matemática precisa de las órbitas planetarias, que proporcionó el rigor matemático necesario al modelo heliocéntrico. Sus aportaciones incrementaron espectacularmente el conocimiento de los científicos sobre el movimiento planetario.

Isaac Newton (1642-1727), matemático y físico británico, empleó los trabajos de Kepler para formular su teoría de la gravitación universal. De acuerdo con la primera ley de Kepler los planetas giran alrededor del Sol en órbitas elípticas en las que el Sol ocupa uno de los focos de la elipse. La segunda ley formula que las áreas barridas por el radio vector que une el centro del planeta con el centro del Sol son iguales en lapsos iguales; como consecuencia, cuanto más cerca está el planeta del Sol con más rapidez se mueve.

Las unidades temáticas que se despliegan en este módulo, tienen carácter teórico-práctico, pues contribuyen a lograr una interrelación apropiada entre la dirección racional y emocional del comportamiento de los estudiantes, desarrollan rasgos del carácter y hábitos del pensar, estimulan la movilidad de los procesos del pensamiento, favorecen la coherencia y precisión al expresar una idea del lenguaje común al matemático y viceversa y capacitan para la valoración crítica del trabajo, tanto propio como de sus condiscípulos.

Es importante, desde la definición de las diferentes expresiones algebraicas que se definen de cada lugar geométrico, contextualizar el contenido con los procesos vitales-escolares y propiciar la lectura de conceptos y relaciones de las propiedades de las figuras planas con las demás ciencias. A partir del estudio de las ecuaciones de rectas y curvas de segundo grado, se desarrollan habilidades en el trabajo con variables, así como en las propiedades y fórmulas de cálculo con las diferentes curvas de segundo grado.

Page 40: repositorio.unesum.edu.ecrepositorio.unesum.edu.ec/bitstream/53000/2123/1/LIBRO-DIP-039.pdf · Quedan todos los derechos reservados. Esta publicación no puede ser reproducida, distribuida,

40

a) consideramos un punto yxP ; cualquiera de la recta;

b) sustituimos las coordenadas de P y de un punto conocido, al

igual que el valor de la pendiente en la fórmula de la misma.

c) efectuamos y expresamos en la forma .

2.2 Geometría plana

Diseñar el círculo despliega la información de los cuadros Augusto

2.3 Fórmulas básicas:

Ecuación cartesiana de la recta:

Ecuación cartesiana de la recta

Para obtener la ecuación cartesiana de la recta seguimos el algoritmo:

Si conocemos un punto y la pendiente:

Formulas básicas

Distancia entre dos puntos

Pendiente de una recta

Condiciones de paralelismo y

perpendicularidad

Coordenadas del punto

medio de un segmento

Dados dos puntos del plano

);( 11 yxP y );( 22 yxQ la distancia

entre ellos está dada por:

2

21

2

21; yyxxQPd

Sean );( 11 yxP y

);( 22 yxQ dos puntos

de una recta no paralelas

al eje “y”; la pendiente:

12

12

xx

yym

Es la tangente del ángulo

que forma la recta con el

semieje “x” positivo (

tanm ).

Sean las rectas r1 y r2 de

pendiente m1 y m2

respectivamente, se cumple que:

a) r1 r2 si y solo si m1 = m2.

b) r1 r2 si y solo si m1 m2 =

-1.

Sea AB un segmento

cuyos extremos tienen

coordenadas );( aa yxA

y

);( bb yxB entonces las

coordenadas del punto

medio M de AB son:

Page 41: repositorio.unesum.edu.ecrepositorio.unesum.edu.ec/bitstream/53000/2123/1/LIBRO-DIP-039.pdf · Quedan todos los derechos reservados. Esta publicación no puede ser reproducida, distribuida,

41

Si conocemos dos puntos:

Hallamos el valor de la pendiente y seguimos el procedimiento anterior utilizando uno de los puntos dados.

Distancia de un punto a una recta

La distancia de un punto 00 ; yxP

a la recta r de ecuación 0 CByAx , se denota d (P;r) y es:

22

00;

BA

CByAxrPd

2.4 Intersección entre rectas y curvas

Para determinar la posición relativa de una recta y una curva de segundo grado (circunferencia, parábola, elipse o hipérbola), se despeja una variable en la ecuación lineal y se sustituye en la cuadrática, obteniéndose una ecuación de segundo grado de una variable.

Si el discriminante D de esta ecuación cumple que:

D 0: la recta es secante.

D 0: la recta es tangente.

D 0: la recta es exterior.

Para determinar los puntos de intersección, si existen, se resuelve completamente el sistema de ecuaciones formado.

2.5 Ejemplos

Ejemplo 1

Sean M (–1; –2) y N (7; 2) los vértices de un triángulo isósceles MNP de base MN .

a) Indique cuál de los siguientes pares ordenados pueden ser las coordenadas punto P: (7; 9), (3; –2) y (–1; 8).

b) Calcula el área del MNP.

Ejemplo 2

Sean A (0; –4), B (5; –5), C (6; 0) y D (1; 1) los vértices de un cuadrilátero. Demuestra que es un cuadrado y calcula su área.

Solución:

1. Como el triángulo es isósceles de base MN , entonces debemos probar que los lados NPMP .

Por lo tanto, se tiene que calcular la distancia de los puntos dados hasta los indicados, con el punto (7; 9) se tiene:

uyyxxPNd

yyxxPMd

749490)²92()²77()²()²();(

18512164)²92()²71()²()²();(

1212

1212

Esto quiere decir que no corresponde al primer punto, probemos ahora con el segundo punto (3; -2)

Page 42: repositorio.unesum.edu.ecrepositorio.unesum.edu.ec/bitstream/53000/2123/1/LIBRO-DIP-039.pdf · Quedan todos los derechos reservados. Esta publicación no puede ser reproducida, distribuida,

42

uyyxxPNd

uyyxxPMd

24321616)²22()²37()²()²();(

416016)²22()²31()²()²();(

1212

1212

Como los valores encontrados no son iguales entonces, este no corresponde al punto P, solo nos queda probar con el punto (-1; 8).

uyyxxPNd

uyyxxPMd

101003664)²82()²17()²()²();(

101001000)²82()²11()²()²();(

1212

1212

Y como ambas distancias son iguales entonces el punto es P (-1; 8)

b) Consideremos la siguiente figura de análisis

2

hMNAMNP

Pero: uMN 54801664)²22()²17(

.

L (3; 0) por ser punto medio de MN

uLPdh 54806416)²08()²31);(

Sustituyendo en el área obtenemos:

2

hMNAMNP

= ²40

2

80

2

5454u

2. Basta probar que las diagonales son iguales y perpendiculares.

Efectivamente: A (0; –4), B (5; –5), C (6; 0) y D (1; 1)

uDBd

uCAd

134523616)²15()²15();(

134521636)²04()²60();(

Con esto se prueba que las diagonales son iguales, ahora se debe probar que son perpendiculares:

2

3

4

6

51

51

3

2

6

4

06

40

12

12

BD

AC

m

xx

yym

y

Al multiplicar estas pendientes se obtiene (-1) por lo que se concluye que las diagonales son perpendiculares

Por tanto, el cuadrilátero es un cuadrado por tener sus diagonales iguales y perpendiculares.

Page 43: repositorio.unesum.edu.ecrepositorio.unesum.edu.ec/bitstream/53000/2123/1/LIBRO-DIP-039.pdf · Quedan todos los derechos reservados. Esta publicación no puede ser reproducida, distribuida,

43

2.6 Ecuaciones y características de la circunferencia

Revise el siguiente video sugerido y participe en el foro identificando la importancia del teorema de Pitágoras para la resolución de las ecuaciones de la circunferencia.

Una circunferencia es el conjunto de puntos del plano que equidistan de un punto fijo llamado centro. La distancia de cualquier punto al centro es el radio. Notación: C(O; r)

d (P; O) = r

x2 + y2 = r2 Ecuación canónica es aquella que tiene su centro en el origen de coordenadas

√(x – h)² + (y – k)² = r

Si elevamos al cuadrado ambos miembros se obtiene (x – h) ² + (y – k) ² = r², la cual representa la ecuación cartesiana de la circunferencia en el plano.

2.7 Ejemplos

Ejemplo 1

Escribe la ecuación de la circunferencia que tiene centro O y radio r.

O (3;8), r = 4

Solución: (x – 3)2 + (y – 8)2= 16

b) O (–4;6), r = 2√2

Solución: (x +4)2 + (y –6)2= (2√2)² = 4.2= 8

(x +4)2 + (y –6)2= 8

Ejemplo 2

Las siguientes ecuaciones representan circunferencias. Determina su centro y radio.

a) X2 + y2 = 2 b) (x – 2)2 + (y + 7)2 = 25

Soluciones

a) Centro O (0; 0) radio r = √2 b) Centro O (2; -7) radio r = 5

Page 44: repositorio.unesum.edu.ecrepositorio.unesum.edu.ec/bitstream/53000/2123/1/LIBRO-DIP-039.pdf · Quedan todos los derechos reservados. Esta publicación no puede ser reproducida, distribuida,

44

Ejemplo 3

Halla la ecuación de la circunferencia de centro O (– 4; –1) y que pasa por el punto P (1; 2).

Solución

Sabemos que: r = d (P; O)

r= √(xP − xO)2 + (yP − yO)2

r = √ (1 + 4)² + (2 + 1)²

r = √25 + 9 = √34

Por tanto la ecuación es (x +4)² + (y +1)² = 34

Ejemplo 4

Los extremos de un diámetro de una circunferencia son los puntos P (6; –2) y

Q (–2; 4). Escribe su ecuación.

Solución: Sea O: punto medio de PQ, entonces al calcular el centro O tenemos:

O (xP+ xQ

2;

yP+ yQ

2)

O (6−2

2;

−2+4

2) De donde se obtiene que el centro es: O (2; 1), luego para hallar el radio calculamos la

distancia entre el punto P y el centro O, obteniéndose lo siguiente:

√(x – h)² + (y – k)² = r

√(6 – 2)² + (−2 – 1)² = r

√(4)² + (– 3)² = r

√25 = r

r = 5

Luego la ecuación cartesiana es: (x – 2)2 + (y– 1)2 = 25

2.8 Foro. - Ejercicio propuestos

Una vez que haya realizado los ejercicios propuestos participe en el foro indicado el método que uso para resolver los problemas

Ejercicios propuestos

Dado el cuadrilátero ABCD cuyos vértices son: A (– 2; y), B (1; 4), C (6; – 1), D (3; – 4)

a) ¿Para qué valor de “y” el cuadrilátero es un paralelogramo? Clasifícalo.

b) Escribe la ecuación de la recta que contiene a la mediana del BCD respecto al lado BC.

c) Escribe la ecuación de la circunferencia que circunscribe dicho cuadrilátero.

Page 45: repositorio.unesum.edu.ecrepositorio.unesum.edu.ec/bitstream/53000/2123/1/LIBRO-DIP-039.pdf · Quedan todos los derechos reservados. Esta publicación no puede ser reproducida, distribuida,

45

2.9 Ecuaciones y características de la parábola

La parábola es el lugar geométrico de los puntos de un plano que equidistan de un punto fijo del plano llamado foco, y de una recta fija llamada directriz, que no pasa por el foco.

En ambas figuras se cumple que:

d(P;F) = d(P;l)

d(F;l) = 2 p Parámetro

d(F;V) = d(V;l) = p Semiparámetro.

En la representación gráfica de la parábola pueden ocurrir los siguientes casos.

Page 46: repositorio.unesum.edu.ecrepositorio.unesum.edu.ec/bitstream/53000/2123/1/LIBRO-DIP-039.pdf · Quedan todos los derechos reservados. Esta publicación no puede ser reproducida, distribuida,

46

Primer caso: Parábola con vértice en el origen que abre hacia arriba. Ecuación x² = 4py

Segundo caso: Parábola con vértice en el origen que abre hacia abajo. Ecuación x²= - 4py

Tercer caso: Parábola con vértice en el origen que abre hacia la derecha. Ecuación y²= 4px

Cuarto caso: Parábola con vértice en el origen que abre hacia la izquierda. Ecuación y²= - 4px

2.10 Ecuación canónica de la parábola

Para todos los casos se ha representado la forma canónica de la ecuación de la parábola, es decir con vértice V (0; 0)

Ejemplos

Ejercicios resueltos con esta condición:

Representa gráficamente y escribe la ecuación de la parábola que cumple:

a) V (0;0), F (2;0) b) F(0; –1 ) , l: y =1

a) Soluciones a) V (0;0), F (2;0)

Como la curva tiene vértice en el origen de coordenadas y abre hacia la derecha, entonce su ecuación es de la forma: y2 = 4px y al ser valor p = 2, entonces la ecuación es y2 = 8x

b) F(0; –1 ) , l: y =1

Page 47: repositorio.unesum.edu.ecrepositorio.unesum.edu.ec/bitstream/53000/2123/1/LIBRO-DIP-039.pdf · Quedan todos los derechos reservados. Esta publicación no puede ser reproducida, distribuida,

47

Como el valor de p = 1 y es una parábola que abre hacia abajo, entonces la ecuación es:

x2 = −4y

2.11 Ecuación de la parábola de vértice V(h;k)

Análisis de casos

Primer caso: abre hacia arriba (x – h)2 = 4p(y – k)

Segundo caso: abre hacia abajo (x – h)2 = – 4p(y – k)

Tercer caso: abre hacia la derecha (y – k)2 = 4p(x – h)

Cuarto caso: abre hacia la izquierda (y – k)2 = –4p(x – h)

Ejemplo

Ejercicio resuelto

Representación gráficamente y escribe la ecuación de la parábola que cumple:

a) V(2;3) ; F(2;5)

b) F(2; –2) ; l: y – 4 = 0

c) V(3;1) ; l: x – 6 = 0

a) V(2; 3) y F(2; 5) Se trata de una parábola que abre hacia arriba

La misma tiene como ecuación (x – 2)² = 8(y – 3)

b) F(2; –2) ; l: y – 4 = 0

Se trata de una ecuación que abre hacia abajo, cuyo gráfico es:

Page 48: repositorio.unesum.edu.ecrepositorio.unesum.edu.ec/bitstream/53000/2123/1/LIBRO-DIP-039.pdf · Quedan todos los derechos reservados. Esta publicación no puede ser reproducida, distribuida,

48

El mismo tiene como ecuación (x – 2)² = - 12 (y – 1)

c) V(3;1) ; l: x – 6 = 0

Se trata de una parábola que abre hacia la izquierda, la cual tiene como gráfico el siguiente:

La misma tiene como ecuación (y – 1)² = - 12(x – 3)

2.12 Ecuaciones y características de la elipse

La elipse es el lugar geométrico de los puntos de un plano tales que la suma de sus distancias a dos puntos fijos del plano, llamados focos, es una constante mayor que la distancia entre los focos.

En la elipse cumple las siguientes características:

d (P; F1) + d (P; F2) =2 a k > d (F1; F2)

F1F2̅̅ ̅̅ ̅̅ ̅ = 2c; OF1 ̅̅ ̅̅ ̅̅ = OF2̅̅ ̅̅ ̅= c (Distancia y semidistancia focal)

A1A2̅̅ ̅̅ ̅̅ ̅= 2a; OA1̅̅ ̅̅ ̅̅ = OA2̅̅ ̅̅ ̅̅ = a (Distancia y semidistancia entre los vértices principales)

B1B2̅̅ ̅̅ ̅̅ ̅ = 2b; OB1 ̅̅ ̅̅ ̅̅ = OB2̅̅ ̅̅ ̅̅ = b (Distancia y semidistancia entre los vértices no principales)

Por lo anterior podemos plantear que

2 a: representa la longitud del eje mayor

2 b: representa la longitud del eje menor

1 c: representa la longitud entre los focos.

Page 49: repositorio.unesum.edu.ecrepositorio.unesum.edu.ec/bitstream/53000/2123/1/LIBRO-DIP-039.pdf · Quedan todos los derechos reservados. Esta publicación no puede ser reproducida, distribuida,

49

Triángulo característico: a2 = b2 + c2

Excentricidad: e =c

a<1

Elipse con centro en el origen de coordenadas de ecuación

a²+

b²= 1 Elipse de eje principal sobre el eje x.

b²+

a²= 1 Elipse de eje principal sobre el eje y.

Si la elipse se traslada su centro para un punto (h; k), entonces se obtiene la ecuación cartesiana de la

elipse (x−h)²

a²+

(y−k)²

b²= 1

2.13 Ecuaciones y características de la hipérbola

Se llama hipérbola al lugar geométrico de los puntos para los cuales la diferencia de sus distancias a dos puntos fijos del plano, llamados focos, es una cantidad constante; la diferencia indicada se toma en su valor absoluto y suele designarse por 2 a: Los focos de la hipérbola se designan con las letras F1 y F2 y la distancia entre ellos con 2c.

Según la definición de la hipérbola 2 a < 2c o a <c

Si los ejes del sistema de coordenadas cartesiano rectangular se han elegido de manera que los focos de la hipérbola se sitúan en el eje de abscisas, simétricamente con respecto al origen de coordenadas la ecuación de la hipérbola de este sistema de coordenadas es de la forma:

a²−

b²= 1 (Hipérbola con centro en el origen y eje principal sobre el eje x.)

a²−

b²= 1 (Hipérbola con centro en el origen y eje principal sobre el eje y)

Gráficamente se representa de la siguiente manera.

Page 50: repositorio.unesum.edu.ecrepositorio.unesum.edu.ec/bitstream/53000/2123/1/LIBRO-DIP-039.pdf · Quedan todos los derechos reservados. Esta publicación no puede ser reproducida, distribuida,

50

De la misma conocemos las características siguientes:

Centro O (0; 0) es de ecuación de la forma x²

a²−

b²= 1 Esta ecuación reciben el nombre de ecuación

canónica de la hipérbola.

Vértices A1 (-a; 0) y A2 (a; 0)

Focos F1 (.c; 0) y F2 (c; 0)

Triángulo característico c2= a2 + b2

Excentricidad: e = c

a; e 1

Si la hipérbola traslada su centro para un punto (h; k), entonces al igual que las cónicas anteriores obtenemos la ecuación cartesiana de la hipérbola.

(x − h)²

a²−

(y − k)²

b²= 1

Ejemplo

Ejercicios resueltos.

1. Escribe la ecuación de la hipérbola que cumple: O (3;2) , a = 12u y e = 1,083

Solución

Como e = c

a, entonces c = a. e

c = 12. 1,083 = 12,996 ≈ 13

Del triángulo característico c² = a² + b², sustituyendo los valores de a y c se obtiene que:

b² = c² - a² = 13² - 12² = 1669 – 144 = 25, por tanto, b = 5 u

Luego podemos obtener las siguientes ecuaciones:

I) (x−3)²

144−

(y−2)²

25= 𝟏Eje principal paralelo al eje x.

II) (y−2)²

25−

(x−3)²

144= 𝟏 Eje principal paralelo al eje y.

2. Determina centro, vértices, focos y excentricidad de la hipérbola

9x2 –16y2 –72x + 96y –144 = 0

Page 51: repositorio.unesum.edu.ecrepositorio.unesum.edu.ec/bitstream/53000/2123/1/LIBRO-DIP-039.pdf · Quedan todos los derechos reservados. Esta publicación no puede ser reproducida, distribuida,

51

Solución:

9x2 –16y2 –72x + 96y –144 = 0

9x² - 72 x – 16y² + 96y = 144

9 (x² - 8x) – 16 (y² - 6y) = 144

9(x² - 8x + 16) – 16(y² - 6x + 9) = 144 +144 - 144

9(x – 4)² - 16(y – 3)² = 144 /: 144

(𝐱 − 𝟒)²

𝟏𝟔−

(𝐲 − 𝟑)²

𝟗= 𝟏

La cual representa una hipérbola con centro en O (4; 3) y eje principal paralelo al eje x.

De acuerdo a la ecuación se obtiene que a = 4; b = 3 y c = 5, luego podemos obtener los siguientes puntos:

Vértices A1 (0;3) y A2(8;3)

Focos F1 (–1;3) y F2(9;3)

Excentricidad: e = c

a=

5

4= 1,25

Otra característica importante a valorar en la hipérbola es la determinación de sus asíntotas tal y como se ilustra en la siguiente figura:

c

Consideremos a b, obteniéndose como asíntotas las siguientes: y = b

a x y y = −

b

a x

X a -a

Y b -b

𝐲 = 𝐛

𝐚 𝐱 𝐲 =

𝐛

𝐚 𝐱

Las rectas que se aproximan a la curva y que no llegan tocarla reciben el nombre de asíntotas.

X A -a

Y -b -b

Page 52: repositorio.unesum.edu.ecrepositorio.unesum.edu.ec/bitstream/53000/2123/1/LIBRO-DIP-039.pdf · Quedan todos los derechos reservados. Esta publicación no puede ser reproducida, distribuida,

52

2.14 Aplicación

A lo largo de toda la historia del desarrollo de la ciencia y la técnica las ecuaciones de las rectas y las curvas de segundo grado han provocado constantemente la atención de muchos investigadores científicos. Esto se debe a que la elipse, la hipérbola y la parábola son muy frecuentes en los fenómenos de la naturaleza y de la actividad humana que nos rodea. Demos por ejemplo algunos casos donde las mismas se utilizan:

Una piedra o proyectil lanzados bajo un ángulo respecto al horizonte, vuela por una curva próxima a la parábola.

Para construir diversos proyectores y antenas se utilizan los llamados espejos parabólicos.

En la producción de muchas industrias de mecanismos de piñones elípticos.

Hay que destacar que son muchos los empleos que se tienen de las relaciones estudiadas en esta unidad y que son muy importantes en la formación de diferentes profesionales, además, por el uso que tienen en la astronáutica y en el movimiento que se realiza de nuestro planeta alrededor del Sol, actualmente cuando en torno a la Tierra giran por las órbitas elípticas millare4s de satélites artificiales, cuando han sido enviadas a la Luna, Venus, y Marte decenas de estaciones cósmicas, las curvas de segundo grado se utilizan aún más intensamente que antes.

2.15 Ejercicios resueltos

Ejercicio 1

1. Sean M (–1; –2) y N (7; 2) los vértices de un triángulo isósceles MNP de base MN . a) Indique cuál de los siguientes pares ordenados pueden ser las coordenadas punto P: (7; 9),

(3; –2) y (–1; 8).

b) Calcula el área del MNP. 2. Sean A (0; –4), B (5; –5), C (6; 0) y D (1; 1) los vértices de un cuadrilátero. Demuestra que es un

cuadrado y calcula su área.

Solución

1. Como el triángulo es isósceles de base MN , entonces debemos probar que los lados NPMP .

Por lo tanto, se tiene que calcular la distancia de los puntos dados hasta los indicados, con el punto (7; 9) se tiene:

uyyxxPNd

yyxxPMd

749490)²92()²77()²()²();(

18512164)²92()²71()²()²();(

1212

1212

Esto quiere decir que no corresponde al primer punto, probemos ahora con el segundo punto (3; -2)

uyyxxPNd

uyyxxPMd

24321616)²22()²37()²()²();(

416016)²22()²31()²()²();(

1212

1212

Como los valores encontrados no son iguales entonces, este no corresponde al punto P, solo nos queda probar con el punto (-1; 8).

Page 53: repositorio.unesum.edu.ecrepositorio.unesum.edu.ec/bitstream/53000/2123/1/LIBRO-DIP-039.pdf · Quedan todos los derechos reservados. Esta publicación no puede ser reproducida, distribuida,

53

uyyxxPNd

uyyxxPMd

101003664)²82()²17()²()²();(

101001000)²82()²11()²()²();(

1212

1212

Y como ambas distancias son iguales entonces el punto es P (-1; 8)

b) Consideremos la siguiente figura de análisis

2

hMNAMNP

Pero: uMN 54801664)²22()²17(

.

L (3; 0) por ser punto medio de MN

uLPdh 54806416)²08()²31);(

Sustituyendo en el área obtenemos:

2

hMNAMNP

= ²40

2

80

2

5454u

Basta probar que las diagonales son iguales y perpendiculares.

Efectivamente: A (0; –4), B (5; –5), C (6; 0) y D (1; 1)

uDBd

uCAd

134523616)²15()²15();(

134521636)²04()²60();(

Con esto se prueba que las diagonales son iguales, ahora se debe probar que son perpendiculares:

2

3

4

6

51

51

3

2

6

4

06

40

12

12

BD

AC

m

xx

yym

y

Al multiplicar estas pendientes se obtiene (-1) por lo que se concluye que las diagonales son perpendiculares

Por tanto, el cuadrilátero es un cuadrado por tener sus diagonales iguales y perpendiculares.

2.16 Evaluación de la sección

1) Dados los puntos A (2,2); B (5,6); C (13,0) y D(x, y) vértices de un cuadrilátero.

1.1 Se puede afirmar que el triángulo ABC es:

a) ___ Acutángulo

Page 54: repositorio.unesum.edu.ecrepositorio.unesum.edu.ec/bitstream/53000/2123/1/LIBRO-DIP-039.pdf · Quedan todos los derechos reservados. Esta publicación no puede ser reproducida, distribuida,

54

b) ____ Obtusángulo c) ____ Rectángulo

1.2 Una ecuación de una recta paralela al lado AB que pasa por el punto C es:

a) ____ 3x + 2y – 5 = 0 b) ____ 4x – 3y – 52 = 0 c) ____ 3x + 4y – 39 = 0

1.3 Las coordenadas del punto D para que el cuadrilátero ABCD (en ese orden) sea un paralelogramo son:

a) ____ D (3; 4) b) ____ D (10; -4) c) ____ D ( 10; 4)

2) Sean M (2,0); N (6,3) y P (3,7) vértices de un triángulo.

2.1 Las coordenadas del punto medio del lado NP̅̅ ̅̅

a) ____ (6; 5) b) ____ (4,5; 5) c) ____ (3; 6)

2.2 El perímetro del triángulo MNP es de:

a) _____ 12 u b) ____ 6,5 u c) ____ 17,05 u

3. Sean A (-2,-1); B (1,3) y C (5,0) los vértices de un triángulo.

3.1 Una ecuación de la mediatriz relativa al lado AB̅̅ ̅̅ es: (-0,5, 1)

a) ____ 4x + 3y – 12 = 0

b) ____ 6x + 8y – 5 = 0

c) ____ 3x – 4y – 8 = 0

3.2 El área del triángulo ABC es de:

a) ___ 10 u² b) ___ 15 u² c) ___ 12,5 u²

4. En una circunferencia los extremos de un diámetro son A (-3,0) y B (4,0), entonces si C(x, 5/2) pertenece a la circunferencia sus coordenadas son:

a) ____ (1/2; 5/2) b) ____ (0; 5/2) c) ____ (3/2; 5/2)

5. Los puntos A (0,-1); B (-2,3); C (2,1) y D (4,-3) (en ese orden) son los vértices de un paralelogramo.

5.1 El paralelogramo ABCD se clasifica como un:

a) ____ cuadrado

Page 55: repositorio.unesum.edu.ecrepositorio.unesum.edu.ec/bitstream/53000/2123/1/LIBRO-DIP-039.pdf · Quedan todos los derechos reservados. Esta publicación no puede ser reproducida, distribuida,

55

b) ____ rectángulo

c) ____ rombo

5.2 La ecuación de la recta que pasa por el punto medio del lado AB y no intercepta a la diagonal AC es:

a) ___ x + y + 2 = 0

b) ___ x – y + 2 = 0

c) ___ x – y – 2 = 0.

Respuestas de los ejercicios y retroalimentación

2.17 Ejercicios sobre circunferencia

1. Dada la ecuación de la circunferencia x2 + y2 + 10x – 6y + 21 = 0 . entonces la ecuación de la circunferencia concéntrica de radio doble del radio anterior es:

a) ___ (x - 5)2 + (y + 3)2 = 52 b) ___ (x + 5)2 + (y – 3)2 = 26 c) ___ (x + 5)2 + (y – 3)2 = 52

Solución y retroalimentación

_X_ (x + 5)2 + (y – 3)2 = 52

2. Halla la longitud de la circunferencia de ecuación x2 + y2 – 8x + 2y + 10 = 0. Determina el área del círculo.

Solución:

X2 + y2 – 8x + 2y + 10 = 0

X2 – 8x + y2 + 2y = –10

X2–8x+16+ y2+2y+1= –10+16+ 1

(x – 4)2 + (y + 1)2 = 7

Luego se puede afirmar que: r = √7 ≈ 2,65 u

LC= 2r 2·3,14·2,65 16,6 u

A= r2 3,14·(√7)²= 3,14·7= 21,98 u2

A 22 u2

1.1c) _X_ Rectángulo 1.2 b) _X_ 4x – 3y – 52 = 0

2.1 b) _X_ (4,5; 5) 2.2 c) _X_ 17,05 u

3.1 b) _X_ 6x + 8y – 5 = 0 3.2 c) _X_ 12,5 u²

4. a) _X_ (1/2; 5/2)

5.1 c) _X_ rombo 5.2 b) _X_ x – y + 2 = 0

Page 56: repositorio.unesum.edu.ecrepositorio.unesum.edu.ec/bitstream/53000/2123/1/LIBRO-DIP-039.pdf · Quedan todos los derechos reservados. Esta publicación no puede ser reproducida, distribuida,

56

2.18 Ejercicios sobre la ecuación de la parábola

1. Dada la ecuación de la parábola y2 + 2y – 16x – 47 = 0. Determine su posición, coordenadas del vértice, coordenadas del foco y ecuación de la directriz.

1.1 Se puede afirmar que la parábola:

a) ___ abre hacia arriba

b) ___ abre hacia la derecha

c) ___ abre hacia la izquierda

1.2 Tiene como vértice al punto:

a) ___ V (3; -1) b) ___ V (3; 1) c) ____ (-3; -1)

1.3 Las coordenadas del foco son_

a) ____ F (1; 2) b) ___ F (1; - 1) c) ____ F (-1; -1)

1.4 La ecuación de la directriz está dada por:

a) ___ l: x + 7 = 0 b) ____ l: x - 7 = 0 c) ____ l: x + 3 = 0

Solución y retroalimentación:

1.1 b) _X_ abre hacia la derecha 1.2 c) _X_ (-3; -1)

1.3 b) _X_ F (1; - 1) 1.4 a) _X_ l: x + 7 = 0

2.19 Ejercicios sobre elipse

1. La ecuación de elipse con eje mayor paralelo al eje x que cumple las condiciones siguientes Centro O (3; 2), a = 13 y e = 0,923 es:

a) ___ (x−3)²

25+

(y−2)²

169= 1

b) ___ (x−3)²

25+

(y−2)²

169= 1

c) _____ (x−2)²

169+

(y−3)²

25= 1

2. Dada la ecuación siguiente de la elipse 9x2+25y2–72x–150y+144 = 0, entonces:

2.1 La elipse tiene como centro al punto

a) ____ O (.4; 3) b) ____ O (4; 3) c) ____ O (-4; -3)

2.2 La elipse tiene como vértices principales a los puntos:

a) ___ A1 (-1; 3) y A2 (9; 3) b) ___ A1 (1; -3) y A2 (9; -3) c) ___ A1 (1; 3) y A2 (7; 3)

2.3 Las coordenadas de los focos de la elipse son:

a) ____ F1 (4; 6) y F2 (4; 0) b) ___ F1 (0; 3) y F2 (8; 3) c) ___ F1 (0; 0) y F2 (6; 3)

Solución y retroalimentación:

c) _X__ (x−2)²

169+

(y−3)²

25= 1 2.1 b) _X_ O (4; 3)

a) _X_ A1 (-1; 3) y A2 (9; 3) 2.3 ___ b) ___ F1 (0; 3) y F2 (8; 3)

Page 57: repositorio.unesum.edu.ecrepositorio.unesum.edu.ec/bitstream/53000/2123/1/LIBRO-DIP-039.pdf · Quedan todos los derechos reservados. Esta publicación no puede ser reproducida, distribuida,

57

3. La siguiente ecuación 9x2 + y2 – 8y + 7 = 0 de ella podemos señalar lo siguiente:

Tiene como posición

a) ___ Elipse de centro O (0; 4) y eje mayor sobre el eje y. b) ___ Elipse de centro O (0; 4) y eje mayor sobre el eje x. c) ___ Elipse con centro en el origen y eje mayor sobre el eje y

Las coordenadas de los vértices no principales son:

a) ____ B1 (-1; -4) y B2 (-1; 0) b) ___ B1 (-1; 4) y B2 (1; 4) c) ___ B1 (0; 0) y B2 (0; 3)

Solución y retroalimentación:

a) _X__ Elipse de centro O (0; 4) y eje mayor sobre el eje y.

b) _X___ B1 (-1; 4) y B2 (1; 4)

4. En una elipse el eje menor es igual al semieje mayor y la distancia focal es de 12 u entonces la ecuación de esa elipse sabiendo que está centrada en el punto O (4; 5) con eje mayor paralelo al eje X es:

___ (x−4)²

12+

(y−5)²

48= 1

___ (x−4)²

48+

(y−5)²

12= 1

___ (x−4)²

18+

(y−5)²

16= 1

Solución y retroalimentación:

_X_ (x−4)²

48+

(y−5)²

12= 1

2.20 Ejercicios sobre hipérbola

1. Dadas la ecuación siguiente de una hipérbola 9y² - 16x² = 144, entonces:

1.1 La posición de la misma es:

a) ___ Hipérbola de centro O (0; 0) y eje mayor sobre el eje y. b) ___ Hipérbola de centro O (0; 0) y eje mayor sobre el eje x. c) ___ Hipérbola con centro O (9; 16) y eje mayor paralelo al eje y

1.2 La longitud del eje principal 2a es igual a:

a) ___ 12 u b) ___ 6 u c) ___ 8 u

1.3 La excentricidad numérica de la hipérbola es:

a) ___ e = √10

3 b)____ e =

2√10

3 c) ___e =

2

3

Solución y retroalimentación

1.1 a) _X_ Hipérbola de centro O (0; 0) y eje mayor sobre el eje y.

1.2 c) _X_ 8 u 1.3 a) _X_ e = √10

3

Page 58: repositorio.unesum.edu.ecrepositorio.unesum.edu.ec/bitstream/53000/2123/1/LIBRO-DIP-039.pdf · Quedan todos los derechos reservados. Esta publicación no puede ser reproducida, distribuida,

58

2. Si una hipérbola que tiene como asíntotas las rectas

y =4

5 x e y = −

4

5 x + 8 y su eje principal es paralelo al eje x entonces su ecuación es:

___ (x+5)²

25−

(y−4)²

16= 1 b) ___

(x−5)²

25−

(y−4)²

16= 1 c) __

(x+5)²

25−

(y+4)²

16= 1

Solución y retroalimentación

b) _X_ (x−5)²

25−

(y−4)²

16= 1

2.21 Ejercicios integradores de las diferentes curvas de segundo grado

Objetivos: Sistematizar el desarrollo de las habilidades de operar con las ecuaciones y las curvas de segundo grado en la solución de ejercicios integradores.

1. Escribe la ecuación y representa gráficamente la elipse, cuya longitud del eje mayor es la distancia entre los puntos de intersección de la parábola x2=8y y la recta

y – 2 = 0, sabiendo que está centrada en el foco de la parábola y de eje menor 2p.

Solución del ejercicio.

Primeramente, busquemos los puntos de intersección entre las dos curvas y la recta, es decir resolver el sistema de ecuaciones siguiente:

x² = 8y

y – 2 = 0

Despejando y en (2) se obtiene que y = 2, luego sustituyendo en (1) x² = 16, de donde se puede afirmar que x = ± 4

Por la ecuación de la parábola x² = 8y, se obtiene 4p = 8, entonces p = 2, luego se obtiene que el vértice es V (0; 0) y el foco es F (0; 2)

Al ser una elipse de eje paralelo al eje X con centro en O (0; 2) y a = 4 y b = 2, entonces la ecuación es

16+

(y−2)²

4= 1; Luego al calcular el valor de c se obtiene que c²= 16 – 4 = 12, es decir c ≈ 3,5 u

Luego obtenemos los vértices y los focos siguientes:

A1(–4; 2), A(4;2); F1(–3,5; 2) y F1(3,5 ; 2)

Logrando de esta manera la gráfica siguiente:

La gráfica es un a elipse con centro en O (0; 2) y eje paralelo al eje X

Page 59: repositorio.unesum.edu.ecrepositorio.unesum.edu.ec/bitstream/53000/2123/1/LIBRO-DIP-039.pdf · Quedan todos los derechos reservados. Esta publicación no puede ser reproducida, distribuida,

59

2. Escribe la ecuación de la circunferencia que los extremos de un diámetro son el vértice y el foco de la parábola (y – 1)2 = 24(x + 5).

Solución:

Por la ecuación de la parábola se obtiene que V (-5; 1) y al ser 4p = 24, entonces p = 6, luego el foco es el punto F (1; 1) al determinar el centro de la circunferencia, primeramente buscamos el valor del radio el cual está dado por la relación r = ½ p = 3 u

Al buscar el centro de la circunferencia al hallar el punto medio entre los puntos dados tenemos lo siguiente:

O (xV+ xF

2;

yV+ yF

2) = (

−5+ 1

2;

1+ 1F

2) = (−2; 1)

Luego la ecuación es: (x + 2)² + (y – 1)² = 9

Page 60: repositorio.unesum.edu.ecrepositorio.unesum.edu.ec/bitstream/53000/2123/1/LIBRO-DIP-039.pdf · Quedan todos los derechos reservados. Esta publicación no puede ser reproducida, distribuida,

60

Page 61: repositorio.unesum.edu.ecrepositorio.unesum.edu.ec/bitstream/53000/2123/1/LIBRO-DIP-039.pdf · Quedan todos los derechos reservados. Esta publicación no puede ser reproducida, distribuida,

61

CAPÍTULO III. NÚMEROS COMPLEJOS

3.1 ¿QUÉ SON LOS NÚMEROS COMPLEJOS?

Desde el estudio de los diferentes dominios numéricos, habrás valorado la necesidad de su ampliación, y que sucesivamente han tenido que realizarse para poder resolver las operaciones fundamentales de cálculo; así vemos que al estudiar las ecuaciones cuadráticas de la forma ax² + bx + c = 0 consideramos que la misma no tiene solución cuando el discriminante D = b² - 4ac es negativo, pues las raíces cuadradas de números negativos no están definidas en el conjunto de los números reales.

También existen fórmulas de resolución de ecuaciones cúbicas. Estas fórmulas de solución fueron estudiadas en la escuela media, su obtención se debió al trabajo de los matemáticos italianos Escipione del Ferro, Nicolo Fontano (Tartaglia) y Gerónimo de Cardano. Entre los siglos XV y XVI. En estas fórmulas aparecen de nuevo raíces cuadradas de números negativos, pero estos matemáticos también rechazaron esas soluciones y no lograron darse cuenta de que al rechazar las raíces están excluyendo soluciones reales de la ecuación dada.

Esta particularidad es valorada de igual forma por el italiano Rafael Bombelli (1530 – 1579) al cual se le debe la primera noción del número complejo según se puede apreciar en su libro Álgebra editado en 1572 en el cual utiliza la fórmula de Tartaglia para la solución de ecuaciones de tercer grado, donde considera además una forma para expresar las raíces cuadradas de números negativos, a los cuales denotó números imaginarios.

Los números imaginarios, entre los matemáticos de la época, no tuvieron una franca acogida, por lo que se tuvieron envueltos por más de 200 años en el más complejo misticismo, esto justifica la denominación de entonces “números imaginarios”

Aún en el siglo XVIII la expresión √−1 estaba desprovista de sentido y representaba un símbolo de una operación imposible que permita establecer vínculos o relaciones imprevistas, que más tarde fueron demostradas por los matemáticos franceses A. Demoivre (1667 – 1754) y J.D`Alembert (1717 – 1783)

y el suizo I. Euler (1707 – 1783), este último introduce la letra i para denotar el símbolo √−1 que en electricidad se representa por i.

Fue en el siglo XIX cuando se da un concepto claro de los números imaginarios, contribuyendo en primer término, a la interpretación geométrica que casi dieron simultáneamente el matemático alemán K.F. Gauss (1777 – 1855), el noruego G. Wessel (1745 – 1818) y el suizo A. J. Argaud (1768 – 1822), en segundo término, una concepción puramente formal de los números complejos que los reduce a números reales.

Este módulo aporta los conocimientos matemáticos para entender que los números complejos, no solo constituyen un complemento en la construcción de los dominios numéricos, sino también son una herramienta de trabajo, imprescindibles en numerosas ramas de la ciencia y la técnica, en el estudio de diferentes temas de Física, como son el movimiento vibratorio, las oscilaciones armónicas y otros fenómenos ondulatorios.

Las unidades temáticas que se despliegan en este módulo, tienen carácter teórico-práctico, pues contribuyen a lograr una interrelación apropiada entre la dirección racional y emocional del comportamiento de los estudiantes, desarrollan rasgos del carácter y hábitos del pensar, estimulan la movilidad de los procesos del pensamiento, favorecen la coherencia y precisión al expresar una idea del lenguaje común al matemático y viceversa y capacitan para la valoración crítica del trabajo, tanto propio como de sus condiscípulos.

Los objetivos de este tema son:

Representar números complejos en todas sus formas matemáticas.

Page 62: repositorio.unesum.edu.ecrepositorio.unesum.edu.ec/bitstream/53000/2123/1/LIBRO-DIP-039.pdf · Quedan todos los derechos reservados. Esta publicación no puede ser reproducida, distribuida,

62

Realizar operaciones con los números complejos con lenguaje y simbología matemática.

Aplicar números complejos en la solución de problemas de las ciencias e ingenierías. Definición de números complejos

3.2 Definición de números complejos

Definición: Un número complejo es un par ordenado de números reales cualesquiera representados en la forma (a; b).

Ejemplos de números complejos: (2; 5); (-4; -3); (2; 0); (5; ); (1

2; √2)

A los números reales que integran el par ordenado se les llama primer y segundo componente del número complejo respectivamente. Un número complejo cuyo segundo componente sea cero se puede considerar como un número real, ya que su representación gráfica pertenece al eje de las x.

Si un número complejo no es real, es decir si el segundo componente es distinto de cero, entonces se dice que el número complejo se denomina imaginario y en el caso que la primera componente sea cero se conoce con el nombre de imaginario puro.

Son números imaginarios los siguientes:

(3; -4); (-5; ); (−1

2; −√2) ; (4; 2); son imaginarios puros: (0; 1); (0; -3); (0;√3); entre otros.

La forma (a; b), siendo a y b números reales, se le llama forma aritmética del número complejo.

Como los números complejos se representan sobre un sistema de coordenadas rectangulares, dicho sistema se le da el nombre de plano complejo y al número imaginario puro (0; 1), se denomina unidad imaginaria y se denota por i.

La representación aritmética (a; b) se puede escribir también de la siguiente forma a + bi, la cual se conoce con el nombre de forma binómica (rectangular) de los números complejos, considerando de esta manera al valor a como la parte real del número complejo y a b como la parte imaginaria del número complejo.

Es importante tener en cuenta que con la representación de los números en estas notaciones es posible resolver las diferentes operaciones que se realizan con los números reales, para esto se considera la siguiente igualdad como artificio, la cual permite hallar las raíces cuadradas de los números negativos, o sea i² = - 1, lo que nos permite considerar que:

√−4 = √4 i² = ±2iy así se realiza con todas las raíces cuadradas negativas.

.- Representación geométrica de los números complejos

Los pares ordenados que representan a un número complejo se puede representar en un plano complejo, Los ejes de coordenadas dividen al plano complejo en 4 cuadrantes. A cada par ordenado de números reales corresponde un único punto situado en el plano complejo, lo que permite afirmar que existe correspondencia biunívoca entre los puntos del plano a los cuales le corresponde un único par ordenado de números reales, lo que permite asegurar que esta correspondencia representa una función. La representación gráfica de un número complejo queda situada en cualquiera de los 4 cuadrantes o en el sistema de coordenadas rectangulares.

Page 63: repositorio.unesum.edu.ecrepositorio.unesum.edu.ec/bitstream/53000/2123/1/LIBRO-DIP-039.pdf · Quedan todos los derechos reservados. Esta publicación no puede ser reproducida, distribuida,

63

Cuando un número complejo está representado en un sistema de coordenadas, al punto P que determina la posición que tiene el número complejo, ese punto recibe el nombre de afijo del número complejo. Si representamos el número P(2; 3) en un sistema de coordenadas como se ilustra a continuación, obtenemos la representación geométrica del número complejo.

El eje XX` recibe el nombre de eje real y el eje YY` recibe el nombre de eje imaginario. Si aplicamos al eje real una rotación de 90° el punto (1; 0) se transforma en el punto (0; 1), es decir la unidad real en la unidad imaginaria.

Si unimos el origen de coordenadas con el punto P se obtiene la representación geométrica del número complejo.

El número complejo está representado de esta manera por un

radio vector OP̿̿̿̿ , para el cual podemos determinar su valor, mediante la representación del cálculo de la hipotenusa de un

triángulo rectángulo, o sea que OP̿̿̿̿ = √2² + 3² = √13 u

En sentido general si se tiene un número complejo a + bi, entonces su módulo se puede representar mediante la expresión

ρ = √a² + b²

3.3 Forma trigonométrica del número complejo

Todo número complejo de la forma a + bi al ser representado en el sistema de coordenadas forma un ángulo φ con el semieje positivo de las X, a ese ángulo entre el sentido positivo del eje OX y el vector de posición del número complejo se llama argumento del número complejo, tal y como se muestra a continuación:

Page 64: repositorio.unesum.edu.ecrepositorio.unesum.edu.ec/bitstream/53000/2123/1/LIBRO-DIP-039.pdf · Quedan todos los derechos reservados. Esta publicación no puede ser reproducida, distribuida,

64

La figura representa un triángulo rectángulo de catetos a y b, del cual al aplicar las razones trigonométricas de un triángulo rectángulo para el ángulo φ se obtiene que:

cosφ = a/ρ de donde se obtiene que a = ρ cos φ, de forma análoga se plantea que:

sen φ = b/ρy de aquí se logra que: b = ρ . sen φ sustituyendo a i b en la expresión a + bi se obtiene la forma trigonométrica de un número complejo de la forma siguiente para un número Z, o sea Z = a = ρ cos φ + i ρ . sen φ Si se extrae factor común ρ: Z = ρ ( cos φ + i sen φ) La cual recibe el nombre de forma trigonométrica de un número complejo. Esto quiere decir que para transformar un número en forma binómica para expresarlo en la forma trigonométrica, debemos buscar primero el módulo, luego el valor del ángulo φ.

Veamos los siguientes ejemplos.

Ejemplos

Ejemplo 1

Transformar en forma trigonométrica el número complejo 2 + 2i

Solución:

ρ = √(a² + b²) = √(2² + 2²) = √8 = 2 √2

Como las dos componentes son positivas el ángulo es del primer cuadrante, luego

tan φ = b

a=

2

2= 1 . Lo cual permite afirmar que φ =

π

4 Por lo tanto el número en forma

trigonométrica se expresa como: Z = 2 √2 ( cos 〖π/4 + i sen π/4 )〗 de forma análoga para el

número - 2 + 2i se obtiene: - 2 + 2i = 2 √2 (cos3π

4 + i sen

4 ).

Ejemplo 2

Representar en forma trigonométrica el número - 1 – i √3

Solución:

Primeramente, al buscar ρ = √(a² + b²) = √((−1)^2 + (−√3)²) = √4 = 2

Para determinar el valor del ángulo, tenemos en cuenta que como las dos componentes son negativas, entonces el ángulo es del tercer cuadrante.

tan φ = b/a = (−√3)/(−1) = √3 De donde se puede afirmar entonces que el ángulo φ = 4π/3

Por tanto: - 1 – i √3 = 2 (cos 〖4π/3 + i sen 4π/3 )〗

En determinados ejercicios el valor del ángulo puede expresarse en el sistema sexagesimal y así se

puede plantear también que: - 1 – i √3 = 2 (cos 240° + i sen 240°)

Ejemplo 3

Dado el número complejo Z = 4 (cos 300° + i sen 300°) transformarlo a la forma binómica.

Page 65: repositorio.unesum.edu.ecrepositorio.unesum.edu.ec/bitstream/53000/2123/1/LIBRO-DIP-039.pdf · Quedan todos los derechos reservados. Esta publicación no puede ser reproducida, distribuida,

65

Solución

Para esto aplicando fórmulas de reducción para el ángulo de 300° se obtiene que:

cos 〖300° = cos 〖(360° − 60°) = cos 〖60° = 1/2〗 〗 〗

sen 300° = sen (360° − 60°) = −sen 60° = − √3/2

Al sustituir estos valores en el número dado se obtiene:

Z = 4 (1/2 − √3/2 i ) = 2 − 2√3 i

3.4 Definición dos números complejos

Dos números complejos son iguales si y solo si sus componentes reales son iguales y sus componentes imaginarios son iguales. Si no cumplen lo anterior son desiguales.

De acuerdo con la definición Z1 = a + bi y Z2 = c + di, se puede afirmar que Z1 = Z2 Si se verifica que a = c y b = d.

Los conceptos de mayor qué y menor que no se establecen para los números complejos.

Suma, resta, multiplicación y división de números complejos.

3.5 Adición de números complejos

Definición: Se llama suma de los números complejos Z1 = a + bi y Z2 = c + di al número complejo Z = Z1 + Z2, cuya parte real es la suma de las partes reales de Z1 y Z2 y la parte imaginaria es la suma de las partes imaginarias de Z1 y Z2.

De esta manera obtenemos que Z = Z1 y Z2 = (a + c) + (b + d) i.

Ejemplos resueltos.

a) (2 + 3i) + ( 3 – 2i ) = 5 + i

b) ( 4 - 2√3 i ) + ( −2 − 4√3i ) = 2 − 6√3 i

c) (2√2 + 3√2i) + ( −√2 − 2√2 i) = √2 + √2 i

3.6 Sustracción de números complejos

Definición: Se llama sustracción de los números complejos Z1 = a + bi y Z2 = c + di al número complejo Z = Z1 – Z2, cuya parte real es la diferencia de las partes reales de Z1 y Z2 y la parte imaginaria es la diferencia de las partes imaginarias de Z1 y Z2.

De esta manera obtenemos que Z = Z1 – Z2 = (a - c) + (b - d) i.

Ejemplos resueltos.

a) (2 + 3i) - ( 3 – 2i ) = -1 + 5 i

b) ( 4 - 2√3 i ) − ( −2 − 4√3i ) = 6 + 2√3 i

c) (2√2 + 3√2i) − ( −√2 − 2√2 i) = 3√2 + 5√2 i

Page 66: repositorio.unesum.edu.ecrepositorio.unesum.edu.ec/bitstream/53000/2123/1/LIBRO-DIP-039.pdf · Quedan todos los derechos reservados. Esta publicación no puede ser reproducida, distribuida,

66

3.7 Multiplicación de números complejos

Dos números complejos Z1 = a + bi y Z2 = c + di se multiplican según la regla ordinaria del producto de polinomios, considerando además la condición de que por convenio i² = - 1 y se separa la parte real de la imaginaria.

Por ejemplo Z1 . Z2 = (a +bi) (c + di) = ac + a.di + bci + bd.i² = a.c + (a.d + bc)i – bd

= (a.c – b.d) + (a.d + b.c) i

Ejemplos resueltos.

a) (2 + 3i) ( 3 – 2i ) = 6 – 4i + 9i – 6i² = 6 + 5i + 6 = 12 + 5i

b) ( 4 – 2√3 i )( −2 − 4√3i ) = −8 − 16√3 i + 4√3 i + 8.3. i² = −8 − 12√3 i − 24 =

−32 − 12√3 i

c) (2√(2 ) + 3√2 i)( −√2 − 2√2 i) = −4 − 8 i − 6i − 12 i² = 8 − 14 i

3.8 División de números complejos

Primeramente, veamos el concepto de números complejos conjugados, es decir si Z = a + bi es un número complejo, entonces el número Z ̅ = a − bi recibe el nombre de número complejo conjugado de Z.

Para dos números complejos conjugados se cumplen las siguientes condiciones:

│Z│ = │Z ̅│

Z + Z ̅ = 2a Es un número real.

Z - Z ̅ = 2bi Es un número imaginario puro.

Z ∙ Z ̅ = (a + bi)(a − bi) = a² − b²i² = a² + b² ya que i² = -1

Se llama cociente de la división de dos números complejos a + bi y c + di al número complejo

x + ´yi que multiplicado por el divisor nos da el dividendo, es decir:

a + bi = (c + di) (x + yi) efectuando este producto se obtiene que

a + bi = cx – dy + (cy + dx)i . De la condición de igualdad de números complejos se obtiene el siguiente sistema de ecuaciones:

(1) cx – dy = a

(2) dx + cy = b

Al resolver este sistema de ecuaciones hallamos que:

x = ac+bd

c²+d² ; y =

cb−ad

c2+d2

Por lo tanto

a + bi

c + di=

ac + bd

c² + d²+

cb − ad

c² + d²i

En lo adelante seguiremos la siguiente regla según aparece en el ejemplo resuelto:

Page 67: repositorio.unesum.edu.ecrepositorio.unesum.edu.ec/bitstream/53000/2123/1/LIBRO-DIP-039.pdf · Quedan todos los derechos reservados. Esta publicación no puede ser reproducida, distribuida,

67

1) 2+3i

2−i=

(2+3i)(2+i)

(2−i)(2+i)=

4+2i+6i+3i²

2²−i²=

1+8i

5=

1

5+

8

5 i

2) 3−√3i

√3+i=

(3−√3i)(√3−i)

(√3+i)(√3−i)=

3√3 − 3i−3i+ √3i²

3+1=

2√3− 6i

4=

√3

2+

3

2 i

3.9 Potencias y raíces de Números Complejos

La elevación de un número complejo a una potencia entera positiva se realiza por la regla de potenciación de un binomio, puesto que es un caso particular del producto de factores complejos iguales, tal y como aparece en los siguientes ejemplos:

( a + bi)² = a² + 2abi + b²i² = a² + 2abi – b² = (a² - b²) + 2abi.

(a + bi)³ = a³ + 3 a²bi + 3ab²i² + b²i³ = a² + 3 a²bi + 3ab² + b³ i = (a³ + 3ab²) + (3ª²b – b³)i

3.9.1 Extracción de la raíz cuadrada de un número complejo

Supongamos que se quiere extraer la raíz cuadrad del número complejo a + bi. Quiere decir hallar un número complejo x + yi tal que su cuadrado sea igual a: a + bi.

Tendremos que:

√a + bi = x + yi ( x; y R)

Elevando al cuadrado ambos miembros se obtiene

a + bi = (x2 − y2) + 2xyi Por la igualdad de números complejos se forma el sistema

x² - y² = a

2xy = b

Resolviendo este sistema de ecuaciones, despejando en la ecuación (2) y = b

2x, sustituyendo este

valor en (1)

x² − b²

4x²= a, Multiplicando esta ecuación por 4x² se forma la ecuación bicuadrada

4x4 − 4ax² − b² = 0

La misma al aplicar la fórmula general de las ecuaciones cuadráticas tiene como solución

x² = a ± √a²+b²

2Como √a² + b² ≥ a, ante el radical hay que tomar el signo positivo para que x² sea un

número positivo o cero; por lo tanto, de x² = a+ √a²+b²

2 se obtiene que:

x = ±√a+ √a2+b2

2; Sustituyendo este resultado en (1) obtenemos que:

y² = −a+ √a²+b²

2 Al determinar los valores de y se obtiene que:

y = ±√−a+ √a2+b2

2 De esta forma se obtienen las raíces cuadradas del número complejo.

Page 68: repositorio.unesum.edu.ecrepositorio.unesum.edu.ec/bitstream/53000/2123/1/LIBRO-DIP-039.pdf · Quedan todos los derechos reservados. Esta publicación no puede ser reproducida, distribuida,

68

La ecuación 2xy = b demuestra que el producto xy tiene el mismo signo que b. Por lo tanto, si b 0; entonces x e y tienen signos iguales; si b < 0, entonces x e y tienen signos diferentes. En la práctica estas fórmulas no se utilizan sino se realiza el paso dado de los cálculos de x e y en cada caso separado.

Ejemplo resuelto:

√1 + 2i = x + yi elevando al cuadrado ambos miembros

1 + 2i = x² - y² + 2xyi Por la igualdad de números complejos formamos el sistema.

x² - y² = 1

xy = 1 despejando y en esta ecuación se obtiene (3) y = 1

x sustituyendo este valor en la ecuación (1)

x² − 1

x²= 1 Multiplicando por x²

x4 − x² − 1 = 0 De donde se obtiene que x² = 1+ √5

2 Lo que permite afirmar que

x = ±√1 + √5

2

Sustituyendo x² en (1) se obtiene que y² = −1+ √5

2 Por tanto y = ±√−1+ √5

2

Finalmente planteamos que:

√1 + 2i = x + yi = ± (√1 + √5

2+ i√

−1 + √5

2)

3.10 Operaciones de los números complejos dados en forma trigonométrica

Sean los números complejos Z1 = ρ1(cos φ1 + isenφ1) y Z2 = ρ2(cos φ2 + isenφ2) Obtendremos que:

Z1 ∙ Z2 = ρ1(cos φ1 + isenφ1) ∙ ρ2(cos φ2 + isenφ2) De este producto multiplicando primeramente los módulos

= (ρ1cos φ1 + ρ1 isenφ1) ∙ (ρ2 cos φ2 + ρ2 isenφ2) - Multiplicando los dos binomios y extrayendo factor común se logra que:

ρ1 ∙ ρ2 (cos φ1 ∙ cos φ2 − senφ1 ∙ senφ2) + ρ1 ∙ ρ2i (cos φ1 ∙ sen φ2 + senφ1 ∙ cosφ2) Aplicando las identidades trigonométricas de la suma de ángulos.

Z1 ∙ Z2 = ρ1 ∙ ρ2[cos(φ1 + φ2) + i sen (φ1 + φ2)] La cual representa la vía de la multiplicación de dos números complejos.

De forma análoga para la división se obtiene que:

Z1

Z2= ρ1 ∙ ρ2[cos(φ1 − φ2) + i sen (φ1 − φ2)]

Con la potenciación se procede como lo siguiente:

Zn = [ρ (cos φ + i sen φ)]n = ρn(cos φ + i sen φ)n

Con la potencia del módulo no existe ningún problema al calcular:

Page 69: repositorio.unesum.edu.ecrepositorio.unesum.edu.ec/bitstream/53000/2123/1/LIBRO-DIP-039.pdf · Quedan todos los derechos reservados. Esta publicación no puede ser reproducida, distribuida,

69

(cos φ + i sen φ)n Se obtiene que:

(cos φ + i sen φ)n = cos nφ + i sen nφ Esta fórmula se conoce con el nombre de fórmula de Moivre.

En particular para n = 2 se obtiene que (cos φ + i sen φ)2 = cos 2φ + i sen 2φ

3.11 Forma exponencial de un número complejo

En diferentes partes del estudio de la matemática, así como en sus aplicaciones (electrónica, radiotécnica, hidráulica, etc.) se utiliza la forma exponencial del número complejo, basada en la fórmula de Euler que relaciona las funciones trigonométricas del argumento real con la función exponencial de argumento imaginario. Veamos la primera fórmula de Euler:

eφi = cos φ + i sen φ Donde el número e tomado como base de los logaritmos naturales es un número irracional (e ≈ 2,718…) este número es tan importante en la matemática como el

número

Si en la fórmula Z = (cos φ + i sen φ) sustituimos la expresión cos φ + i sen φ por eφi

obtendremos Z = eφi La cual recibe el nombre de forma exponencial del número complejo.

Sustituyendo en la fórmula de Euler φ por (-φ) obtendremos la segunda fórmula de Euler

e−φi = cos(−φ) + i sen(− φ) = cos φ − i sen φ

Ejemplo resuelto. Representar en forma exponencial el número complejo Z = √3 + i

Calculemos primero el módulo:

= √a2 + b2 = √3 + 1 = 2

Hallamos el argumento en la relación

tanφ = b

a=

1

√3=

√3

3 ; Entonces φ =

π

6 por lo tanto

Z = √3 + i = eπ

6i

Ejemplo 2 Z = cosπ

2+ isen

π

2= e

π

2i

De las fórmulas (1) y (2) de Euler sumando miembro a miembro se obtiene

eφi + e−φi = 2 cos φ, de donde cos φ = eφi+ e−φi

2 la cual se conoce como la fórmula (3) de Euler.

Si restamos miembro a miembro obtenemos de forma análoga la fórmula (4), o sea.

sen φ = eφi − e−φi

2

3.12 Aplicaciones

Los aspectos teóricos estudiados en esta unidad de los Números Complejos, son necesarios para vincular los contenidos esenciales con las diferentes situaciones problemáticas que permitan constatar algunas de las numerosas aplicaciones de estos números. Por otra parte se ha observado a través de las diferentes temáticas tratadas en esta unidad que los números complejos permiten encontrar las soluciones de ecuaciones cuando estas no son solución del dominio de los números reales; es importante considerar que los números complejos son indispensables en el estudio de la Electricidad General, la Electrotecnia y la Electrónica, para caracterizar mediante la representación permanente de

Page 70: repositorio.unesum.edu.ecrepositorio.unesum.edu.ec/bitstream/53000/2123/1/LIBRO-DIP-039.pdf · Quedan todos los derechos reservados. Esta publicación no puede ser reproducida, distribuida,

70

sinusoide de la Mecánica de los sólidos y de los fluidos, las resonancias y vibraciones y en la determinación de la resistencia de materiales.

El empleo de estos números está cada día más extendido en la enseñanza de las ingenierías técnicas y resulta imposible predecir en qué variados campos pueden surgir aplicaciones. Los números complejos también resultan indispensables para el estudio de la Matemática en niveles superiores, los cuales sirven de base para el tratado de las funciones de variables complejas y de las transformaciones correspondientes.

Como resultado de aprendizaje de este módulo, el estudiante será capaz de interpretar las representaciones gráficas de los números complejos, resolver operaciones de cálculo con los números complejos, así como transformar de la forma binómica a la trigonométrica o viceversa.

3.13 Subsección. - Evaluación

1. Al Calcular:

1.1 (1 + 4i) + (-2 – 2i) Se obtiene:

a) ___ 2 + 4i b) ____ -1 + 2i c) ___ -2 + 2i

1.2 (2√3 - 2i) – (√3 − 3i) Es igual a:

a) ___ 2√3-5i b) ____ √3 + i c) ___ 3√3 + i

1.3 2(cosπ

3− i sen

π

3) + (3 − 4√3 )Es igual a:

a) ___ -3 + 3i√3 b) ____ 3 - 3i√3c) ___ 4 - 5√3 i

1.4 2(cos3π

4+ i sen

4) − 4 (cos

π

4− i sen

π

4) Es igual a:

a) ___ -3√2 + 3i√2b) ____ 3√2- 3i√2c) ___ 3i√2

Solución

Solución y retroalimentación

1.1 b) _X_ -1 + 2i 1.2 b) _X_ √3 + i

1.3 c) _X_ 4 - 5√3 i 1.4 a) _X_ -3√2 + 3i√2

2. Calcular los siguientes productos. 2.1 (3 – 5i) (4 – i) Es igual a: a) ___ 12 + 5i b) ___ 17 – 17i c) ____ 17 + 17i

2.2 (√3 − i√2)(√3 + i√2) Es igual a:

a) ___ 5i b) ___ 5 c) ____ 2√3 − 2i√2

2.3 (√5 + 2 i) (2√5 − 2 i ) Es igual a:

a) ___ 14 + 2i b) ___ 5 - 2i√5 c) ____ 14 + 2i√5

2.4 3 (cos2π

3+ i sen

3) . 2 (cos

π

6+ i sen

π

6)Es igual a:

a) ___ 6 (cos5π

6+ isen

6) b) ___ 6 (cos

π

6+ isen

π

6) c) ____ 6 (cos

6+ isen

6)

2.5 . (cos3π

2+ i sen

2) . 2 (cos

π

3+ i sen

π

3)Es igual a:

Page 71: repositorio.unesum.edu.ecrepositorio.unesum.edu.ec/bitstream/53000/2123/1/LIBRO-DIP-039.pdf · Quedan todos los derechos reservados. Esta publicación no puede ser reproducida, distribuida,

71

a) ___ 2 (cos5π

6+ isen

6) b) ___ 2 (cos

6+ isen

6) c) ____ 2 (cos

11π

6+ isen

11π

6)

Solución y retroalimentación

2.1 c) _X_ 17 + 17i 2.2 b) _X_ 5 2.3 c) _X__ 14 + 2i√5

2.4 a) _X_ 6 (cos5π

6+ isen

6) 2.5 c) __X_ 2 (cos

11π

6+ isen

11π

6)

3. Al calcular los cocientes siguientes se obtiene como resultado_

3.1 1+i

1−i

a) ___ 1 + i b) ____ 1 – i c) ____ i

3.2 1+i√3

1−i√3

a) ___ −1

2+

1

2i√3 b) ____

1

2−

1

2i√3 c) ____ 1 + i√3

3.3 17−6i

3−4i

a) ___ 3 – i b) ___ 3 + 2i c)___ 3 – 4i

3.4 6 (cos5π

6+ i sen

6) : 3 (cos

π

3+ i sen

π

3)

a) ___ 2 (cosπ

2+ isen

π

2) b) ____ 2 (cos

π

3+ isen

π

3) c) ___ 2 (cos

3+ isen

3)

3.5 3 (cos5π

3+ i sen

3): (cos

6+ i sen

6)

a) ___ 3 (cosπ

2+ isen

π

2) b) ____ 3 (cos

6+ isen

6) c) ___ 3 (cos

3+ isen

3)

Solución y retroalimentación

3.1 c) _X_ i 3.2 a) _X_ −1

2+

1

2i√3 3.3 b) _X_ 3 + 2i

3.4 b) _X_ 3 (cos5π

6+ isen

6) 3.5 a) __X_ 2 (cos

π

2+ isen

π

2)

3.14 Ejercicios para calcular

1. Calcular las potencias siguientes y seleccione la respuesta correcta marcando con una X

1.1 (2 − i√2)²

a) ___ 2 + 4i b) ___ 2 - 4i√2 c) ____ 4 + 2i 1.2 (1 + i)³ a) ____ -2 + 2i b) 2 – 2i c) ____ 2 + 2i

2. Extraer la raíz de:

2.1 √21 + 20i a) ____ 5 – 2i b) ____ -5 + 2i c) ____ 5 + 2i Yuraysi Duvergel Cobas

2.2 √5 − 12i a) ____ 3 – 2i b) ____ 3 + 2i c) -3 + 2i Solución y retroalimentación

1.1 b) _X_ 2 - 4i√2 1.2 a) _X_ -2 + 2i 2.1 c)_X_ 5 + 2i 2.2 b) _X_ 3 + 2i

Page 72: repositorio.unesum.edu.ecrepositorio.unesum.edu.ec/bitstream/53000/2123/1/LIBRO-DIP-039.pdf · Quedan todos los derechos reservados. Esta publicación no puede ser reproducida, distribuida,

72

3. Representar en forma exponencial los siguientes números complejos. a) 1 – i

b) -√3 + i c) 2i

4. Transforma en la forma algebraica los siguientes números complejos.

a) 2 eπ

4i

b) e2 + i

c) 2e1-π

4i

3.15 Respuesta de los ejercicios propuestos en las actividades de aprendizaje

a) Para el número complejo 1 – i se tiene que su módulo es ρ = √1 + 1 = √2 Al determinar el ángulo

tenemos tan φ = b

a =

−1

1= −1 Como el ángulo es del cuarto cuadrante al tener el número signo (+; -)

entonces φ = 7π

4 Por tanto 1 – i = √2e

4i

b) Para el número – √3 + i se tiene que ρ = √3 + 1 = √4 = 2 Al calcular el ángulo se obtiene

tan φ = b

a=

1

−√3= −

√3

3 Al tener el número signos (-; +), entonces el ángulo es del segundo

cuadrante, de donde se puede afirmar que φ = 5π

6 . Por lo tanto para el número dado se cumple lo

siguiente: – √3 + i = 2e5π

6i

c) Para el valor 2i se cumple que ρ = √0 + 4 = 2 y como el número es imaginario puro, entonces φ =

π

2 Por lo tanto 2i = 2e

π

2i

Soluciones:

a) 2eπ

4i = 2 (cos

π

4+ i sen

π

4) = 2 (

√2

2+ i

√2

2) = √2 + i√2

b)e2+i = e². ei = e²(cos 1 + isen 1) = 7,4(0,54 + i 0,84) = 3,996 + i 6,216

c)211−i π

4 = 2. e. e−iπ

4 = 2e [cos (−π

4) + i sen (−

π

4)] = 2e (

√2

2− i

√2

2) = e√2 − i√2

Page 73: repositorio.unesum.edu.ecrepositorio.unesum.edu.ec/bitstream/53000/2123/1/LIBRO-DIP-039.pdf · Quedan todos los derechos reservados. Esta publicación no puede ser reproducida, distribuida,

73

CAPITULO IV. MATRICES, SISTEMAS LINEALES Y NO LINEALES

4.1 INTRODUCCIÓN

En este módulo se desarrolla la Unidad 4 sobre MATRICES, SISTEMAS LINEALES Y NO LINEALES. Con ella se logra como resultados de aprendizaje que el estudiante: aplica matrices y determinantes en la solución de sistemas lineales y no lineales.

El desarrollo de los diferentes epígrafes, permiten al estudiante desarrollar capacidades y destrezas a partir de la definición de matrices, su clasificación y propiedades y las operaciones de cálculo básico entre ellas; además de estudiar otras operaciones como la Potencia de una matriz.

Por otra parte, se estudian los Determinantes; Determinante de una matriz, Métodos para encontrar la determinante de una matriz y los Sistemas de Ecuaciones Lineales y no lineales, sus Métodos de solución; con énfasis en la resolución de problemas de sistemas de ecuaciones lineales en dos variables y de inecuaciones en dos variables.

Se profundiza en los métodos para resolver ecuaciones de primer grado y sistemas de ecuaciones lineales, los cuales se pueden representar en forma matricial. En esta Unidad, al profundizar en el estudio de los diferentes tipos de matrices; el cálculo de determinantes y sus propiedades, es conveniente la determinación de la matriz inversa, y la resolución de sistemas de ecuaciones lineales por el método de Gauss, Gauss Jordan y la regla de Cramer.

El estudio de las ecuaciones no lineales, incluye las cuadráticas, las cuales permitirán al estudiante involucrarse con el análisis de la parábola y su aplicación en las Ciencias Sociales, la Economía, la Administración de Empresas y áreas afines.

Podrá calcular los cortes de la curva con el eje de las abscisas y mediante el análisis de su discriminante determinar la naturaleza de las raíces. La solución de sistemas de ecuaciones de segundo grado, le permitirá adquirir las destrezas para resolver ejercicios de Cónicas en Geometría Analítica, tales como los que aparecen en el Módulo V de esta Área.

Los objetivos que se quiere alcanzar con el estudio de este tema son los siguientes:

Clasificar las matrices e identificar sus características y propiedades.

Resolver operaciones con matrices.

Identificar el determinante de una matriz y resolverlo por varios métodos.

Resolver problemas de sistemas de ecuaciones lineales.

Resolver problemas de sistemas de ecuaciones no lineales.

Resolver problemas de sistemas de inecuaciones de dos variables.

4.2 Concepto de matriz

Siempre que colocamos un elemento en filas y columnas hacemos uso de una estructura matricial.

Por ejemplo, cualquier espectáculo en el que las entradas estén numeradas hace uso de este tipo de estructuras. Lo que se hace es dividir la Platea en filas y columnas. Si en nuestra entrada pone Fila 23, asiento 12 nos está indicando que la butaca está en la fila 23 y columna 12.

Cualquier tabla de las que utilizamos en los editores de texto no deja de ser una matriz, ya que está organizada por filas y columnas.

Page 74: repositorio.unesum.edu.ecrepositorio.unesum.edu.ec/bitstream/53000/2123/1/LIBRO-DIP-039.pdf · Quedan todos los derechos reservados. Esta publicación no puede ser reproducida, distribuida,

74

Por ejemplo, la siguiente tabla tiene 3 filas y 4 columnas. El número que ocupa la fila 2 y columna 4 es el cero:

2 1 5 8

3 2 2 0

2 1 6 4

Para que una tabla sea una matriz representativa de algún objeto matemático basta con que en cada celda pongamos algún valor numérico, le quitemos la cuadrícula y la encerremos entre dos grandes paréntesis:

4612

0223

8512

Y ya tenemos una matriz de las que se utilizan habitualmente en matemáticas.

4.2.1. Tipos de matrices

Una matriz en la que el número de filas es igual al número de columnas se dice que es una matriz cuadrada. Lo es, por ejemplo, la matriz:

612

223

512

y no es cuadrada una matriz como:

223

512

La primera tiene tres filas y tres columnas y se dice que es una matriz 3X3. Se lee “tres por tres”. Para referirnos a la segunda, que tiene dos filas y tres columnas, hablamos de una matriz 2X3, matriz "dos por tres".

De manera que, en general, cuando se habla de una matriz m x n se está haciendo referencia a una matriz que tiene m filas y n columnas. Esta forma de referirse a las matrices no es más que un convenio y podría variar de un autor a otro.

Según esto una matriz m x n será cuadrada cuando m = n

Notación

Las matrices suelen denotarse con letras mayúsculas:

A=

612

223

512

B=

223

512

También se utilizan letras para hacer referencia a los elementos que forman la matriz.

Page 75: repositorio.unesum.edu.ecrepositorio.unesum.edu.ec/bitstream/53000/2123/1/LIBRO-DIP-039.pdf · Quedan todos los derechos reservados. Esta publicación no puede ser reproducida, distribuida,

75

C=

ihg

fed

cba

Se comprende que cuando se trabaja con matrices muy grandes, por ejemplo, de 100x200, es decir con 100 filas y 200 columnas (o más), la utilización de las letras del alfabeto no es práctica, por lo que se recurre a una notación del tipo aij en la que i representa la fila y j representa la columna en la que se encuentra el elemento en cuestión.

Se define la matriz cero como aquella en la que todos sus elementos son 0, independientemente del número de filas y columnas que tenga.

Se dice que dos matrices son iguales cuando los son todos los elementos que la forman.

4.2.2. Propiedades de la suma de matrices

Producto entre escalares y matrices:

Dada una matriz A = (aij) y un número real k£ , se define el producto de un número real por una matriz: a la matriz de la misma dimensión que A, en la que cada elemento está multiplicado por k.

k · A = (k · aij)

Propiedades

1) a · (b · A) = (a · b) · A A Mmxn , a, b

2) a · (A + B) = a · A + a · BA, B Mmxn , a

3)(a + b) · A = a · A + b · A A Mmxn , a, b

1. Interna•La suma de dos matrices de orden m x n es otra matriz dimensión m x n.

2. Asociativa

•A + (B + C) = (A + B) + C

3. Elemento neutro

•A + 0 = A

•Donde O es la matriz nula de la misma dimensión que la matriz A.

4. Elemento opuesto

•A + (−A) = O

•La matriz opuesta es aquella en que todos los elementos están cambiados de signo.

5. Conmutativa

•A + B = B + A

Page 76: repositorio.unesum.edu.ecrepositorio.unesum.edu.ec/bitstream/53000/2123/1/LIBRO-DIP-039.pdf · Quedan todos los derechos reservados. Esta publicación no puede ser reproducida, distribuida,

76

4)1 · A = A A Mmxn

4.2.3. Productos entre matrices

Dos matrices A y B se dicen multiplicables si el número de columnas de A coincide con el número de filas de B.

Am x n x Bn x p = Cm x p

El elemento cij de la matriz producto se obtiene multiplicando cada elemento de la fila i de la matriz A por cada elemento de la columna j de la matriz B y sumándolos.

Ejemplo

PROPIEDADES DEL PRODUCTO DE MATRICES

1. Asociativa:

A · (B · C) = (A · B) · C

2. Elemento neutro:

A · I = A

Donde I es la matriz identidad del mismo orden que la matriz A.

3. Distributiva del producto respecto de la suma:

A · (B + C) = A · B + A · C

4. No es Conmutativa:

A · B ≠ B · A

Ejemplo

Page 77: repositorio.unesum.edu.ecrepositorio.unesum.edu.ec/bitstream/53000/2123/1/LIBRO-DIP-039.pdf · Quedan todos los derechos reservados. Esta publicación no puede ser reproducida, distribuida,

77

Podemos ver que, en este caso, A · B ≠ B · A, de hecho, ni si quiera tienen la misma dimensión, pues A · B ∈ M2x2 y B · A ∈ M3x3.

4.3 Definición de determinante

A cada matriz cuadrada A se le asigna un escalar particular denominado determinante de A, denotado por |A| o por det (A).

Determinante de orden uno |a 11| = a 11

Determinante de orden dos

= a 11 a 22 − a 12 a 21

Determinante de orden tres

=

a11 a22 a33 + a12 a23 a31 + a13 a21 a32 −

− a 13 a22 a31 − a12 a21 a 33 − a11 a23 a32.

4.4 Regla de Sarrus

Los términos con signo + están formados por los elementos de la diagonal principal y los de las diagonales paralelas con su correspondiente vértice opuesto.

Los términos con signo − están formados por los elementos de la diagonal secundaria y los de las diagonales paralelas con su correspondiente vértice opuesto.

Page 78: repositorio.unesum.edu.ecrepositorio.unesum.edu.ec/bitstream/53000/2123/1/LIBRO-DIP-039.pdf · Quedan todos los derechos reservados. Esta publicación no puede ser reproducida, distribuida,

78

Menor complementario

Se llama menor complementario de un elemento aij al valor del determinante de orden n−1 que se obtiene al suprimir en la matriz la fila i y la columna j.

Adjunto

Se llama adjunto del elemento aij al menor complementario anteponiendo:

El signo es + si i+j es par.

El signo es − si i+j es impar.

El valor de un determinante es igual a la suma de productos de los elementos de una línea por sus adjuntos correspondientes.

4.4.1 Determinantes de orden superior a tres

Determinante de orden superior a tres

Consiste en conseguir que una de las líneas del determinante esté formada por elementos nulos, menos uno: el elemento base o pivote, que valdrá 1 ó −1.

Seguiremos los siguientes pasos:

En caso negativo

4.5 Propiedades de los determinantes

Propiedades de los determinantes:

1. Nos fijamos en una línea que contenga el mayor número posible de elementos nulos y operaremos para que uno de los elementos de esa línea sea un 1 ó −1 (operando con alguna línea paralela).

2. Dividiendo la línea por uno de sus elementos, por lo cual deberíamos multiplicar el determinante por dicho elemento para que su valor no varié. Es decir, sacamos factor común en una línea de uno de sus elementos.

3. Tomando como referencia el elemento base, operaremos de modo que todos los elementos de la fila o columna, donde se encuentre, sean ceros.

4. Tomamos el adjunto del elemento base, con lo que obtenemos un determinante de orden inferior en una unidad al original

Si a lgún e lemento del determinante vale la unidad, se e l ige una de las dos l íneas: la f i la o la columna, que cont ienen a d icho e lemento (se debe escoger aquel la que contenga e l mayor número posib le de e lementos nulos) .

Page 79: repositorio.unesum.edu.ecrepositorio.unesum.edu.ec/bitstream/53000/2123/1/LIBRO-DIP-039.pdf · Quedan todos los derechos reservados. Esta publicación no puede ser reproducida, distribuida,

79

4.6 Matriz inversa

Ejemplos de operaciones con determinantes

Ejemplo 1

Demostrar, sin desarrollar, que los siguientes determinantes valen cero:

1. |At|= |A|

2. |A|=0 Si: Posee dos líneas iguales

Posee dos líneas iguales

Todos los elementos de una línea son nulos y los elementos de una línea son combinación lineal de las otras

3. Un determinante triangular es igual al producto de los elementos de la diagonal principal.

4. Si en un determinante se cambian entre sí dos líneas paralelas su determinante cambia de signo

5.- Si a los elementos de una línea se le suman los elementos de otra paralela multiplicados previamente por un numeros real el valor del determinante

6.- Si se multiplica un determinante por un número real, queda multiliplicado por dicho número líneal ,pero solo una vez .

7.- Si todos los elementos de una fila o columna están formados por dos sumandos, dicho determinante se descompone en la suma de dos determinantes

8. |A·B| =|A|·|B|

Page 80: repositorio.unesum.edu.ecrepositorio.unesum.edu.ec/bitstream/53000/2123/1/LIBRO-DIP-039.pdf · Quedan todos los derechos reservados. Esta publicación no puede ser reproducida, distribuida,

80

Tiene dos líneas proporcionales.

La tercera columna es igual a la suma de las otras dos.

Ejemplo 2

Sabiendo que |A|=5, calcula los otros determinantes.

4.7 Problemas de aplicación

1. El dueño de un bar ha comprado refrescos, cervezas y vino por importe de $500.00 (sin impuestos). El valor del vino es $60.00 menos que el de los refrescos y de la cerveza conjuntamente. Teniendo en cuenta que los refrescos deben pagar un IVA del 6%, por la cerveza del 12% y por El vino del 30%, lo que hace que la factura total con impuestos sea de $592.4, calcular la cantidad invertida en cada tipo de bebida.

x = Importe en € de los refrescos. x=$120.00

y = Importe en € de la cerveza. y=$160.00 z = Importe en € del vino. z=$220.00

Page 81: repositorio.unesum.edu.ecrepositorio.unesum.edu.ec/bitstream/53000/2123/1/LIBRO-DIP-039.pdf · Quedan todos los derechos reservados. Esta publicación no puede ser reproducida, distribuida,

81

4.7.1. Sistemas de Ecuaciones Lineales

Definición

Llamamos sistemas de dos ecuaciones lineales con dos variables a dos ecuaciones de este tipo que pueden reducirse a la forma:

y que las soluciones de los sistemas de dos ecuaciones con dos variables son las soluciones comunes a las dos ecuaciones que lo forman, es decir, son los pares ordenados (x; y) que satisfacen a ambas ecuaciones.

Métodos de solución

También recordarás que un sistema podía tener una única solución, no tener soluciones, o tener infinitas soluciones

En el próximo módulo estudiarás lo relativo a la función lineal y su representación gráfica en un sistema de coordenadas rectangulares.

Una recta en el plano puede representarse por la ecuación ax + by +c =0 (con x, y R ; a y b no

simultáneamente nulos) despejando y obtenemos , que es precisamente la ecuación de la función lineal.

Si relacionamos las soluciones que tiene un sistema con su interpretación geométrica, concluimos que:

Si el sistema tiene solución única entonces podemos decir que las rectas se intersecan en un punto.

Si el sistema no tiene solución entonces podemos decir que las rectas son paralelas.

Si el sistema tiene infinitas soluciones entonces las rectas coinciden.

¿Recuerdas cómo analizaste sí un sistema tenía una, ninguna o infinitas soluciones?

Teniendo ahora como recurso lo aprendido sobre la función lineal y su representación gráfica, te proponemos un procedimiento mucho más rápido para determinar si un sistema tiene o no solución.

Basta transformar ambas ecuaciones del sistema a la forma y = mx + n y analizar las pendientes de cada una de ellas. Si ellas son iguales, entonces comparamos los términos independientes.

Sea el sistema

Si m1 m2 el sistema tiene una única solución y las rectas se intersecan.

En el caso de ser m1 = m2 y n1 = n2 el sistema tiene infinitas soluciones y las rectas son coincidentes y

en el caso de ser m1 = m2 y n1 n2 el sistema no tiene solución y las rectas son paralelas. Ten presente que para que m1= m2, tiene que ser a1 = a2 y b1 = b2.

0 cy bx a

0c y bx a

222

111

b

cx

b

ay

22

11

nxmy

nxmy

Page 82: repositorio.unesum.edu.ecrepositorio.unesum.edu.ec/bitstream/53000/2123/1/LIBRO-DIP-039.pdf · Quedan todos los derechos reservados. Esta publicación no puede ser reproducida, distribuida,

82

También en este epígrafe aprenderás el método gráfico para resolver sistemas de ecuaciones, el cual consiste en representar gráficamente las rectas cuyas ecuaciones forman el sistema y así podemos determinar las coordenadas de los puntos comunes (en caso que existan).

Por lo general las soluciones que se obtienen por este procedimiento son valores aproximados.

4.7.2. Resolución de sistema de ecuaciones lineales

Resolver sistemas de ecuaciones lineales. (2x2)

Multiplicar convenientemente cada ecuación por un número de modo que al sumar las nuevas ecuaciones obtenidas se elimine una variable y se obtenga el valor de otra.

Sustituir el valor de la variable obtenida en una de las dos ecuaciones para obtener el valor de la otra.

Plantear el conjunto solución.

Resolver sistemas de ecuaciones lineales. (3x3)

Se toman dos parejas de ecuaciones en las que se elimina la misma variable, para obtener dos nuevas ecuaciones con dos variables.

Se resuelve el sistema formado por estas dos ecuaciones.

Se sustituyen los valores encontrados en una de las ecuaciones originales y se halla el valor de la otra variable.

Plantear el conjunto solución.

Ejemplos1

Hallar el conjunto solución del siguiente sistema de ecuaciones

(1) 2x + y – 3z = 5

(2) x – y + 4z = 5

(3) 3x + 2y + 2z = 15

Para darle solución a este sistema empleando los pasos indicados anteriormente, es decir primeramente sumemos las ecuaciones (1) y (2) obteniendo la ecuación (4) 3x + z = 10, luego sumemos

la ecuación (1)2 y la ecuación (3), obteniendo la ecuación (5) 5x + 10z = 25 y dividiendo esta ecuación por 5 se logra (6) x + 2z = 5.

Ahora debemos resolver el sistema de ecuaciones con (4) y (6), es decir:

(4) 3x + z = 10 / (-2) -6x – 2z = - 20

(6) x + 2z = 5 x + 2z = 5 obteniendo -3x = -15; o sea x = 3

Sustituyendo este valor (4) 9 + z = 10, z = 1 y sustituyendo en (1) 6 + y – 3 = 5

y = 2. Se realiza la comprobación y se indica el conjunto solución. S = {(3: 2: 1)}

Ejemplo 2

Resolución de sistemas de ecuaciones lineales de (3x3) por el método de gauss

Sea el siguiente sistema de ecuaciones:

Page 83: repositorio.unesum.edu.ecrepositorio.unesum.edu.ec/bitstream/53000/2123/1/LIBRO-DIP-039.pdf · Quedan todos los derechos reservados. Esta publicación no puede ser reproducida, distribuida,

83

Solución

Algoritmo de solución.

1º Ponemos como primera ecuación la que tenga el coeficiente en x más bajo.

2º Hacemos reducción con la 1ª y 2ª ecuación, para eliminar el término en x de la 2ª ecuación. Después ponemos como segunda ecuación el resultado de la operación:

E'2 = E2 − 3E1

3º Hacemos lo mismo con la ecuación 1ª y 3ª ecuación, para eliminar el término en x.

E'3 = E3 − 5E1

4º Tomamos las ecuaciones 2ª y 3ª, trasformadas, para hacer reducción y eliminar el término en y.

E''3 = E'3 − 2E'2

5º Obtenemos el sistema equivalente escalonado.

6º Encontrar las soluciones.

z = 1

− y + 4 ·1 = −2 y = 6

Page 84: repositorio.unesum.edu.ecrepositorio.unesum.edu.ec/bitstream/53000/2123/1/LIBRO-DIP-039.pdf · Quedan todos los derechos reservados. Esta publicación no puede ser reproducida, distribuida,

84

x + 6 −1 = 1 x = −4

Resolver:

4.8 Sistemas de ecuaciones no lineales

Resolver sistemas de ecuaciones cuadráticas.

Se despeja una variable en la ecuación lineal.

Se sustituye la variable despejada en la ecuación cuadrática.

Page 85: repositorio.unesum.edu.ecrepositorio.unesum.edu.ec/bitstream/53000/2123/1/LIBRO-DIP-039.pdf · Quedan todos los derechos reservados. Esta publicación no puede ser reproducida, distribuida,

85

Se resuelve la ecuación de segundo grado obtenida.

Se calculan los valores de la otra variable.

Plantear el conjunto solución.

Ejercicio resuelto

Determina el conjunto solución del siguiente sistema de ecuaciones:

(1)

(2)

Despejando y en la ecuación (2) se obtiene (3) y = 2x – 2 , sustituyendo (3) en (1)

Multiplicando esta ecuación por (-1), obtenemos y al descomponer en factores se

tiene que ; logrando de esta manera los valores x1 = 2 y x2 = 4, sustituyendo estos valores en (3) se obtiene y1 = 2 y y2 = 6

Se comprueban ambos pares ordenados en el sistema original y se comprueba que ambos son soluciones, por tanto, S = {(2; 2), (4; 6)}

4.8.1 Clasificación de los sistemas de inecuaciones de dos variables

Ante una ecuación pueden ocurrir los siguientes escenarios, que ninguno de los valores de la incógnita arriben a la igualdad, o bien por el contrario, que todo valor posible de la incógnita lo cumpla, en este caso estaríamos ante lo que se denomina en matemáticas identidades y cuando dos expresiones matemáticas coinciden en la desigualdad, a la misma, se la determinará como inecuación.

4.9 Definición de intervalos

En Matemática son de mucha utilidad el empleo de la representación de conjuntos mediante intervalos de números R, que no son más que subconjuntos de números reales, a continuación, ilustramos algunos casos:

4²² yxyx

22 yx

4)²22()22(² xxxx

086²

0448²42²2²

04)48²4(2²2²

xx

xxxxx

xxxxx

086² xx

0)4)(2( xx

Page 86: repositorio.unesum.edu.ecrepositorio.unesum.edu.ec/bitstream/53000/2123/1/LIBRO-DIP-039.pdf · Quedan todos los derechos reservados. Esta publicación no puede ser reproducida, distribuida,

86

1. (a;b) = {x Є R: a < x < b} Intervalo abierto.

Ejemplo: (-4; 6) = {x Є R: -4 < x < 6} ver ilustración:

Figura 9. gráfica 3.1.

2. [a; b] = {x Є R: a < x b} Intervalo cerrado.

Ejemplo: [-5; 3] = {x Є R: -5 < x 3}. Gráficamente:

.

Figura 10: grafica 3.2.

[a; b) = {x Є R: a < x < b} (Intervalo semiabierto o semicerrado)

Ejemplo: [-2; 4) = {x Є R: -2 < x < 4}

Figura 11: grafica 3.3.

2. (a; + ) = {x / x > a} (Intervalo de infinito)

Ejemplo: (1; + ) = {x/ x > 1} Gráficamente se ilustra de la siguiente forma:

Figura 12: grafica 3.4.

Page 87: repositorio.unesum.edu.ecrepositorio.unesum.edu.ec/bitstream/53000/2123/1/LIBRO-DIP-039.pdf · Quedan todos los derechos reservados. Esta publicación no puede ser reproducida, distribuida,

87

3. (- ; a) = {x / x < a} Ejemplo: (- ; 3) = {x / x < 3} Gráficamente:

Figura 13. Gráfica 3.5.

4.10 Resolución de Inecuaciones

Resolver inecuaciones lineales

Los sumandos se transponen de igual manera que en las ecuaciones, agrupando en un miembro los términos que contienen variables y los números en el otro.

El coeficiente de la variable se transpone de igual forma que en las ecuaciones, prestando atención a que:

Si el coeficiente es positivo; la desigualdad no se altera.

Si el coeficiente es negativo; el signo de la desigualdad se invierte.

Resolver inecuaciones cuadráticas

Comparar con cero y reducir términos semejantes.

Multiplicar por (-1) si el coeficiente de la variable de mayor exponente es negativo (cambia el sentido de la desigualdad).

Descomponer en factores.

Determinar los ceros.

Representar los ceros en la recta numérica:

Si es ó entonces se representa con .

Si es ó entonces se representa con .

Representar el signo de cada intervalo. Comenzar con signo (+) y alternarlo con signo (-), siempre de derecha a izquierda. No hay cambio de signo si el cero es doble.

Representar gráficamente el conjunto solución partiendo del último signo del proceso de

resolución de la inecuación (, , , ):

Si es ó se señalan los intervalos con signo (-).

Si es ó se señalan los intervalos con signo (+).

Plantear el conjunto solución:

Si , ,- entonces ( ; ).

Si , entonces ; .

Resolver inecuaciones fraccionarias

Comparar con cero.

Realizar la operación que se obtenga.

Page 88: repositorio.unesum.edu.ecrepositorio.unesum.edu.ec/bitstream/53000/2123/1/LIBRO-DIP-039.pdf · Quedan todos los derechos reservados. Esta publicación no puede ser reproducida, distribuida,

88

Multiplicar por (-1) el numerador o el denominador si el coeficiente de la variable de mayor exponente de uno de ellos es negativo. ( Cambia el sentido de la inecuación )

Descomponer en factores el numerador y el denominador. ( Cuando se simplifican factores hay que excluir del conjunto solución los ceros de los factores que se simplifican)

Representar los ceros en la recta numérica:

Si es ó entonces se representa con .

Si es ó entonces se representa con . Los ceros del denominador nunca se incluyen.

Representar el signo de cada intervalo. Comenzar con signo (+) y alternarlo con signo (-), siempre de derecha a izquierda. No hay cambio de signo si el cero es doble.

Representar gráficamente el conjunto solución partiendo del último signo del proceso de

resolución de la inecuación (, , , ):

Si es ó se señalan los intervalos con signo (-).

Si es ó se señalan los intervalos con signo (+).

Plantear el conjunto solución:

Si , ,- entonces ( ; ).

Si , entonces ; .

4.11 Valor Absoluto

Ejemplos de Ecuaciones e inecuaciones con valor absoluto.

Ejemplo 1

Resuelve las siguientes ecuaciones con valor absoluto:

1) |x| = 4

S = { 4 , - 4 }

Ejemplo 2

2) |3x| = 5

Page 89: repositorio.unesum.edu.ecrepositorio.unesum.edu.ec/bitstream/53000/2123/1/LIBRO-DIP-039.pdf · Quedan todos los derechos reservados. Esta publicación no puede ser reproducida, distribuida,

89

Ejemplo 3

3) |x - 3| = 1

S = {4, 2}

Ejemplo 4

4) |1 + 5x| = - 3

Sabemos que siempre tiene que ser:

|1 + 5x| ≥ 0 ∀x ∈ R

Luego nunca puede ocurrir:

|1 + 5x| = - 3

Por tanto, la ecuación no tiene solución

Ejemplo 5

5) |x + 4| = x + 1

Comprobamos la solución:

Por tanto, la ecuación no tiene solución.

4.12 Ecuaciones cuadráticas

Definición

Una ecuación cuadrática es una ecuación polinómica en la que la mayor potencia de la variable es dos. La forma general de tales ecuaciones en la variable x es

ax2 + bx + c = 0; donde a, b y c son constantes.

Generalmente, existen dos valores de x que pueden satisfacer la ecuación, y son:

Page 90: repositorio.unesum.edu.ecrepositorio.unesum.edu.ec/bitstream/53000/2123/1/LIBRO-DIP-039.pdf · Quedan todos los derechos reservados. Esta publicación no puede ser reproducida, distribuida,

90

Resolución de ecuaciones cuadráticas

Igualar a cero. (Transponer todos los términos a un mismo miembro)

Reducir términos semejantes.

Descomponer en factores.

Igualar a cero cada factor.

Despejar en cada caso y obtener el valor de la variable.

Comprobar.

Plantear el conjunto solución.

Cuando tengas que resolver una ecuación cuadrática, después de expresada en la forma

02 cbxax (a0), si vas a utilizar la fórmula puedes seguir el algoritmo siguiente:

Identifica los coeficientes a, b y c.

Sustituye los valores en la fórmula de la discriminante acbD 42 .

Si D 0, entonces no posee soluciones reales.

Si D = 0, entonces posee una sola solución a

bx

2

Si D 0, entonces posee dos soluciones a

Dbx

22,1

.

4.12.1 Gráfica de una ecuación cuadrática

En las coordenadas cartesianas, la gráfica de una función cuadrática y = ax2 + bx + c es una parábola. Las soluciones X1 y X2 representan los puntos donde la gráfica cruza el eje X. Si la gráfica cruza dos veces el eje, existen dos raíces reales distintas. Si la gráfica toca al eje X en un punto, las dos raíces son iguales. Si la gráfica no cruza el eje X, no existen raíces reales. En este caso, el discriminante es negativo y las raíces son dos números complejos conjugados.

Ejemplos

Ejemplo 1: Análisis de una Ecuación Cuadrática y su gráfica. Función Cuadrática con dos Raíces Reales

Encontrar las raíces de la función f x = x2 + 3 x - 10

Solución: Como las raíces son los valores donde la función es 0, buscamos resolver la ecuación f(x)=0

Factorizando la expresión obtenemos f x = (x+5) ( x-2)

Como el producto anterior es cero, entonces:

Las raíces de la función f x = x2 + 3 x - 10 son x=5 y x=-2. Puedes visualizar estas raíces observando la gráfica de esta función, que es la siguiente:

x+5 = 0 x = -5 o x-2 = 0 x = 2

Page 91: repositorio.unesum.edu.ecrepositorio.unesum.edu.ec/bitstream/53000/2123/1/LIBRO-DIP-039.pdf · Quedan todos los derechos reservados. Esta publicación no puede ser reproducida, distribuida,

91

Si una función cuadrática tiene dos raíces reales su fórmula se puede escribir en la forma k(x-a) (x-b) donde a y b son números reales. En este caso, la gráfica atraviesa el eje x dos veces.

Ejemplo 2: Función Cuadrática con una Raíz Real

Encontrar las raíces de la función f x = x2 - 2 x + 1

Solución:

Como las raíces son los valores donde la función es 0, buscamos resolver la ecuación f(x)=0

Factorizando la expresión obtenemos

f x = x - 1 x - 1

Como f(x)=0, entonces:x - 1 = 0 x = 1

La función f x = x2 - 2 x + 1 tiene una raíz y es x=1

Como vemos en la siguiente figura, la gráfica de la función toca el eje x sin cruzarlo, por lo que sólo tiene una raíz:

Si una función cuadrática tiene una raíz real su fórmula se puede escribir en la forma k(x-a)2 y su gráfica toca el eje x pero no lo cruza.

Page 92: repositorio.unesum.edu.ecrepositorio.unesum.edu.ec/bitstream/53000/2123/1/LIBRO-DIP-039.pdf · Quedan todos los derechos reservados. Esta publicación no puede ser reproducida, distribuida,

92

Ejemplo 3: Ecuaciones cuadráticas con raíces imaginarias. (Función Cuadrática sin Raíces Reales)

Encontrar las raíces de la función f x = 2 x2 - 3 x + 2

Solución: Como las raíces son los valores donde la función es 0, buscamos resolver la ecuación f(x)=0

Cuando vemos que no es muy fácil la factorización podemos recurrir a la fórmula cuadrática. Estudiamos la fórmula cuadrática en la lección ecuaciones cuadráticas.

x = − − 3 ± − 3 2 − 4 2 2 2 2

x = 3 ± 9 − 16 4

x = 3 ± 7 i 4

Las raíces de la función f x = 2 x2 - 3 x + 2 son x = 3 + 7 i 4 y x = 3 - 7 i 4

Cuando la gráfica no intercepta el eje x, las raíces de la función cuadrática son imaginarias, como vemos en la gráfica correspondiente:

Si, usando la fórmula cuadrática, obtenemos raíces imaginarias para función cuadrática, significa que la función no tiene raíces reales. En este caso, la gráfica no toca el eje x.

4.13 Suma y producto de las raíces

Como vimos en los ejemplos previos, existe una relación entre los factores de la función cuadrática y sus raíces.

Si r1 y r2 son raíces de una función cuadrática, entonces ésta se puede escribir en la siguiente forma: f x = k (x – r1) (x – r2 ); donde k ≠ 0 , es un número real .

Como consecuencia, podemos afirmar lo siguiente:

Si r es una raíz de una función cuadrática, entonces existe un factor (x-r) en la fórmula de la función.

Page 93: repositorio.unesum.edu.ecrepositorio.unesum.edu.ec/bitstream/53000/2123/1/LIBRO-DIP-039.pdf · Quedan todos los derechos reservados. Esta publicación no puede ser reproducida, distribuida,

93

Ejemplo 1

Hallar la función cuadrática cuyas raíces son 2 y -1 y cuyo intercepto es -6.

Solución

Por la relación que existe entre las raíces y los factores de la función cuadrática, esta tendrá la forma:

f x = k x - 2 x + 1

Además como el intercepto es -6, significa f(0)=-6. Reemplazando x=0 en la fórmula anterior:

f 0 = k 0 - 2 0 + 1 = - 6

De donde:

- 2 k = - 6

y entonces:

k = 3

Finalmente, la función que buscamos es:

f x = 3 x2 - 2 x + 1

Esta es la gráfica de la función. Puedes verificar que la función tiene las raíces 2 y -1 y su intercepto es -6.

Ejemplo 2

Hallar la función cuadrática cuya única raíz es 1 y cuyo intercepto con el eje de y es 2.

Solución

Por la relación que existe entre las raíces y los factores de la función cuadrática, esta tendrá la forma:

f x = k x - 1 2

Además como el intercepto es 2, significa f(0)=2

Reemplazando x=0 en la fórmula anterior:

f 0 = k 0 - 1 2 = 2

Page 94: repositorio.unesum.edu.ecrepositorio.unesum.edu.ec/bitstream/53000/2123/1/LIBRO-DIP-039.pdf · Quedan todos los derechos reservados. Esta publicación no puede ser reproducida, distribuida,

94

De donde:

k = 2

Finalmente, la función que buscamos es:

f x = 2 x - 1 2

Esta es la gráfica de la función. Puedes verificar que su raíz es 1 y su intercepto es 2.

Ejemplo 3

Hallar la función cuadrática cuyas raíces imaginarias son i y -i y cuyo intercepto con el eje de y es 2.

Solución

Por la relación que existe entre las raíces y los factores de la función cuadrática, esta tendrá la forma:

f x = k x - i x + i ⇒ f x = k x 2 + 1

Además como el intercepto es 2, significa f(0)=2

Reemplazando x=0 en la fórmula anterior:

2 = k 0 2 + 1 ⇒ k = 2

Finalmente, la función que buscamos es:

f x = 2 x - i x + i

Esta es la gráfica de la función. Puedes verificar que no tiene raíces reales y su intercepto es 2.

Page 95: repositorio.unesum.edu.ecrepositorio.unesum.edu.ec/bitstream/53000/2123/1/LIBRO-DIP-039.pdf · Quedan todos los derechos reservados. Esta publicación no puede ser reproducida, distribuida,

95

4.14 Problemas de aplicación

1. Escribe una ecuación de segundo grado cuyas soluciones son: 3 y −2.

SOLUCIÓN: Suma = 3 − 2 = 1

Producto = 3 · (−2) = −6

x2 − x − 6 = 0

2. Determinar k de modo que las dos raíces de la ecuación x2 − kx + 36 = 0 sean iguales.

SOLUCIÓN: b2 − 4ac = 0

K2 − 4 · 36 = 0 k2= 144

3. La suma de dos números es 5 y su producto es −84. Halla dichos números.

SOLUCIÓN: x2 − Sx + P = 0

4. Dentro de 11 años la edad de Pedro será la mitad del cuadrado de la edad que tenía hace 13 años. Calcula la edad de Pedro.

Edad actual x

Edad hace 13 años x − 13

Edad dentro de 11 años x + 11

Edad actual 21

5. Para vallar una finca rectangular de 750 m² se han utilizado 110 m de cerca. Calcula las dimensiones de la finca.

Page 96: repositorio.unesum.edu.ecrepositorio.unesum.edu.ec/bitstream/53000/2123/1/LIBRO-DIP-039.pdf · Quedan todos los derechos reservados. Esta publicación no puede ser reproducida, distribuida,

96

Semiperímetro 55

Base x

Altura 55 − x

x · (55 − x) = 750

x2 − 55x + 750 = 0

x = 25 x = 30

Las dimensiones de la finca son 30 m y 25 m

6. Un jardín rectangular de 50 m de largo por 34 m de ancho está rodeado por un camino de arena uniforme. Halla la anchura de dicho camino si se sabe que su área es 540 m².

(50 + 2x) · (34 + 2x) − 50 · 34 = 540

4x2 + 168x − 540 = 0 x2 + 42x − 135 = 0

x = 3 y x = −45

La anchura del camino es 3 m.

4.15 Sistemas de Ecuaciones Cuadráticos

Métodos de solución

Algoritmo para resolver sistemas de ecuaciones cuadráticas

Se despeja una variable en la ecuación lineal.

Se sustituye la variable despejada en la ecuación cuadrática.

Se resuelve la ecuación de segundo grado obtenida.

Se calculan los valores de la otra variable.

Plantear el conjunto solución.

Page 97: repositorio.unesum.edu.ecrepositorio.unesum.edu.ec/bitstream/53000/2123/1/LIBRO-DIP-039.pdf · Quedan todos los derechos reservados. Esta publicación no puede ser reproducida, distribuida,

97

Ejercicio resuelto

Determina el conjunto solución del siguiente sistema de ecuaciones:

(1) 4²² yxyx

(2) 22 yx

Despejando y en la ecuación (2) se obtiene (3) y = 2x – 2, sustituyendo (3) en (1)

4)²22()22(² xxxx

086²

0448²42²2²

04)48²4(2²2²

xx

xxxxx

xxxxx

Multiplicando esta ecuación por (-1), obtenemos 086² xx y al descomponer en factores se

tiene que 0)4)(2( xx ; logrando de esta manera los valores x1 = 2 y x2 = 4, sustituyendo estos valores en (3) se obtiene y1 = 2 y y2 = 6

Se comprueban ambos pares ordenados en el sistema original y se comprueba que ambos son soluciones, por tanto, S = {(2; 2), (4; 6)}

4.16 Operaciones con matrices

1. Dadas las matrices:

Calcular:

A + B; A − B; A x B; B x A;

SOLUCIONES

Page 98: repositorio.unesum.edu.ecrepositorio.unesum.edu.ec/bitstream/53000/2123/1/LIBRO-DIP-039.pdf · Quedan todos los derechos reservados. Esta publicación no puede ser reproducida, distribuida,

98

4.17 Aplicaciones

Ejercicios de aplicación

1. En un centro deportivo hay 400 atletas varones más que hembras. Se decidió trasladar para otro centro al 70% de los varones y al 20% de las hembras, quedando en el centro inicial 100 hembras más que varones. ¿Cuántos atletas de cada sexo se quedaron en el centro deportivo?

Solución: Designemos con variables lo que se plantea en el problema.

Cantidad de varones…. x

Cantidad de hembras…. y

(1) x = y + 400

(2) 0,3x+100=0,8y

Sustituyendo (1) en (2) yy 8,0100)400(3,0

2205,0

8,01001203,0

y

yy

5,0

220y

440y y al sustituir este valor en (1) se obtiene x = 840

252840

100

30

Cantidad de varones que se quedan

352440

100

80

Cantidad de hembras que se quedan

Respuesta: En el centro deportivo se quedaron 252 varones y 352 hembras.

2. La suma de los valores absolutos de las tres cifras de un número es 14. La cifra de las decenas es igual a la suma de las cifras de las centenas y unidades. La diferencia entre la cifra de las decenas y la cifra de las centenas es 3. ¿Cuál es el número?

Datos:

Cifra de las unidades… u

Cifra de las decenas…. d

Cifra de las centenas… c

Para resolver el problema debemos plantear el siguiente sistema de acuerdo a lo planteado en el texto del mismo.

3

14

cd

ucd

udc

Al sustituir (2) en (1) se obtiene d + d = 14, o sea, 2d =14, de donde d = 7, pero como d – c = 3 y en (2) d – c =u por tanto u = 3 y sustituyendo d = 7 y u = 3 en (1) c = 14 – (d+u) = 14 – 10 = 4. Repuesta: El número es 473.

Page 99: repositorio.unesum.edu.ecrepositorio.unesum.edu.ec/bitstream/53000/2123/1/LIBRO-DIP-039.pdf · Quedan todos los derechos reservados. Esta publicación no puede ser reproducida, distribuida,

99

3. En la aplicación de descomposición factorial, es importante considerar que la diferencia de dos términos, por un trinomio que consta del cuadrado del primer término más el producto de los dos, más el cuadrado del segundo término, es igual a la diferencia de los cubos de los dos términos algebraicos. 3.1. Se trata de demostrar que

2233 yxyxyxyx .

Tendremos:

2 2 3 2 2 2 2 3 3 3x y x xy y x x y xy x y xy y x y

Es decir 3322 yxyxyxyx , tal como queríamos demostrar.

3.2. Ejemplos:

3.2.1. Comprobar que

4228 23 xxxx

Solución:

3.2.2. Comprobar que

2233 91216342764 yxyxyxyx

Solución:

2 2 3 3

3 3

4 3 16 12 9 64 48 36 48 36 27

64 27

x y x xy y x x xy x xy y

x y

3.2.3, Comprobar que

63243296 96432278 bbaababa

Solución:

2 3 4 2 3 6 6 4 3 2 6 4 3 2 6 9

6 9

2 3 4 6 9 8 12 18 12 18 27

8 27

a b a a b b a a b a b a b a b b

a b

4.18 Aplicaciones del algoritmo de solución de ecuaciones lineales a la resolución de problemas

Solución de ecuaciones lineales a la resolución de problemas

Un avión va de una ciudad B a una ciudad A y regresa en 100 minutos. A causa del viento el viaje de ida demora 12 minutos más que el de regreso.

¿Cuántos minutos demora cada viaje?

2 3

3

2 2 4 2 4 2 4 8

8

x x x x x x x x

x

Page 100: repositorio.unesum.edu.ecrepositorio.unesum.edu.ec/bitstream/53000/2123/1/LIBRO-DIP-039.pdf · Quedan todos los derechos reservados. Esta publicación no puede ser reproducida, distribuida,

100

Solución

Identificando las condiciones dadas en el texto y las relaciones entre las partes, seleccionamos los datos:

Tiempo de regreso:

Tiempo de ida:

Tiempo total de ida y regreso: minutos.

Teniendo en cuenta las relaciones de igualdad que se establecen, escribimos la ecuación [1] correspondiente:

Procedemos a resolver la ecuación.

y este es el tiempo que demora el viaje de regreso, pero la ida demora minutos más, por tanto, el viaje de ida demora 56 minutos.

Comprobamos si las sumas de las cantidades obtenidas se corresponden con el total:

Escribimos la respuesta literal:

El viaje de ida demora 56 minutos y el de regreso 44 minutos.

4.19 Aplicaciones utilizando el método de Gauss

1. Un cliente de un supermercado ha pagado un total de 156 dólares por 24 l de leche, 6 kg de jamón serrano y 12 l de aceite de oliva. Calcular el precio de cada artículo, sabiendo que 1 l de aceite cuesta el triple que 1 l de leche y que 1 kg de jamón cuesta igual que 4 l de aceite más 4 l de leche.

SOLUCIÓN

leche x

jamón y

aceite z

leche 1 $

Page 101: repositorio.unesum.edu.ecrepositorio.unesum.edu.ec/bitstream/53000/2123/1/LIBRO-DIP-039.pdf · Quedan todos los derechos reservados. Esta publicación no puede ser reproducida, distribuida,

101

jamón 16 $

aceite 3 $

2. Un videoclub está especializado en películas de tres tipos: infantiles, oeste americano y terror. Se sabe que:

El 60% de las películas infantiles más el 50% de las del oeste representan el 30% del total de las películas.

El 20% de las infantiles más el 60% de las del oeste más del 60% de las de terror al representar la mitad del total de las películas.

Hay 100 películas más del oeste que de infantiles.

Halla el número de películas de cada tipo.

SOLUCIÓN

infantiles x

oeste y

terror z

Sustituimos el valor de y en las dos ecuaciones iniciales y multiplicamos la última obtenida por 3.

infantiles 500 películas

oeste 600 películas

terror 900 películas

4.20 Aplicaciones de las matrices

Una fábrica produce dos modelos de lavadoras, A y B, en tres terminaciones: N, L y S. Produce del modelo A: 400 unidades en la terminación N, 200 unidades en la terminación L y 50 unidades en la terminación S. Produce del modelo B: 300 unidades en la terminación N, 100 unidades en la terminación L y 30 unidades en la terminación S. La terminación N lleva 25 horas de taller y 1 hora de

Page 102: repositorio.unesum.edu.ecrepositorio.unesum.edu.ec/bitstream/53000/2123/1/LIBRO-DIP-039.pdf · Quedan todos los derechos reservados. Esta publicación no puede ser reproducida, distribuida,

102

administración. La terminación L lleva 30 horas de taller y 1.2 horas de administración. La terminación S lleva 33 horas de taller y 1.3 horas de administración.

1. Representar la información en dos matrices. 2. Hallar una matriz que exprese las horas de taller y de administración empleadas para cada uno

de los modelos.

Matriz de producción

Filas: Modelos A y B Columnas: Terminaciones N, L, S

Matriz de coste en horas:

Filas: Terminaciones N, L, S Columnas: Coste en horas: T, A

Matriz que expresa las horas de taller y de administración para cada uno de los modelos:

Resolver; en forma matricial, el sistema:

SOLUCIÓN

Page 103: repositorio.unesum.edu.ecrepositorio.unesum.edu.ec/bitstream/53000/2123/1/LIBRO-DIP-039.pdf · Quedan todos los derechos reservados. Esta publicación no puede ser reproducida, distribuida,

103

4.21 Ejercicios para repaso

1. Comprobar que:

464646 2 xxxx .

Solución: Tendremos

2

2

6 4 6 4 6 4

2 24

x x x x

x x

.

1. Comprobar que:

353535 2 xxxx .

Solución: Tendremos

2

2

5 3 5 3 5 3

8 15

x x x x

x x

.

2. Dados:

babaxRxxPxxM 3264129 22

1423 223 xQxxN

a) Descomponga en factores cada una de las expresiones dadas.

b) Calcule y simplifique: (2x2/N - 1/M): Q/P

Solución:

a) )²23(412²9 xxxM )12)(23(2²6 xxxxP

)13)(()(3 xbababaxR ; )23²(²2³3 xxxxN

)12)(12(1²4 xxxQ

b) (2x2/N - 1/M) : Q/P 2²6

1²4

412²9

1

²2³3

²21²2

xx

x

xxxx

x

P

Q

MN

x

)12(

)23(

)²23²(

²)23²(2

)12)(12(

)12)(23(

)²23(

1

)23²(

²2

x

x

xx

xxx

xx

xx

xxx

x

12

23

)²23²(

)12²(3

12

23

)²23²(

²3³6

12

23

)²23²(

²²4³6

x

x

xx

xx

x

x

xx

xx

x

x

xx

xxx

Al simplificar se

obtiene: 23

3

x

3. Compruebe que:

Page 104: repositorio.unesum.edu.ecrepositorio.unesum.edu.ec/bitstream/53000/2123/1/LIBRO-DIP-039.pdf · Quedan todos los derechos reservados. Esta publicación no puede ser reproducida, distribuida,

104

Si A = 9x2 - 1 B = x2 + x - 20 C = 3x2 + 14x – 5 D = x2 + 10x + 25

Solución:

4. Hallar el conjunto solución de las siguientes inecuaciones:

a) 0)32(

³²5

x

xx

b) xx

x

xx

x

4

32

3²2

9²4

Solución:

a) 0)32(

³²5

x

xx al multiplicar la inecuación por (-1) se obtiene lo siguiente:

032

³²5

x

xx Luego buscando los ceros del numerador (CN) y los ceros del denominador (CD) se

obtiene:

Ceros del numerador: 5x² + x³ = x²(5 + x) de donde se tiene que x1 = 0 es un cero doble x2 = - 5

Cero del denominador: x3 = 2

3

Solución gráfica:

Solución analítica: ;5,15; S

l.q.q.d.

b) xx

x

xx

x

4

32

3²2

9²4)

0)4(32

)1²)(1(

)1)(32(

)32)(32(

x

x

xxx

xx

xx Al simplificar en la operación indicada debemos

tener en cuenta que: 1;2

3x ya que estos valores indefinen a los denominadores obteniendo

entonces:

4

13:

x

x

D

C

B

A

4

13

)5)(13(

)²5(

)4)(5(

)13)(13(

514²3

2510²

20²

1²9

x

x

xx

x

xx

xx

xx

xx

xx

x

C

D

B

A

D

C

B

A

Page 105: repositorio.unesum.edu.ecrepositorio.unesum.edu.ec/bitstream/53000/2123/1/LIBRO-DIP-039.pdf · Quedan todos los derechos reservados. Esta publicación no puede ser reproducida, distribuida,

105

041² xxx ; la cual conduce a la inecuación 032² xx

0)1)(3( xx Esta expresión tiene dos ceros que son: 31 x y 12 x

Solución gráfica:

Solución analítica: 1;3S /

2

3

5. Comprobar que

224236 91216342764 ccbbcbcb

Solución:

2 4 2 2 6 2 2 2 2 2 2 3

6 3

4 3 16 12 9 64 48 36 48 36 27

64 27

b c b b c c b b c b c b c b c c

b c

EJERCICIOS DE ECUACIONES LINEALES

1.

2.

3.

Page 106: repositorio.unesum.edu.ecrepositorio.unesum.edu.ec/bitstream/53000/2123/1/LIBRO-DIP-039.pdf · Quedan todos los derechos reservados. Esta publicación no puede ser reproducida, distribuida,

106

4.

4.22 Resolución de problemas que conducen a Ecuaciones Lineales

Al plantear la ecuación que conduce a la resolución de un problema pueden aparecer algunos elementos con los que has trabajado, como el uso de paréntesis.

1. Si a un número se le adiciona su duplo aumentado en 4753880, se obtiene el mismo resultado que sustraer a 14121997, diez veces el número aumentado en 7452166 ¿Cuál es el número?

Solución

Identificando las condiciones dadas en el texto, seleccionamos los datos:

Número buscado:

Teniendo en cuenta las relaciones que se establecen entre los elementos del problema, planteamos la ecuación:

Procedemos a resolver la ecuación obtenida de la manera siguiente:

Eliminando paréntesis

Agrupando términos semejantes

Reduciendo términos semejantes

Dividiendo por

Page 107: repositorio.unesum.edu.ecrepositorio.unesum.edu.ec/bitstream/53000/2123/1/LIBRO-DIP-039.pdf · Quedan todos los derechos reservados. Esta publicación no puede ser reproducida, distribuida,

107

Escribimos la respuesta literal:

El número es

4.23 Resolución de problemas y ejercicios que conducen a Ecuaciones Cuadráticas

1. Calcula las dimensiones de un rectángulo cuya diagonal mide 75 m, sabiendo que es semejante a otro rectángulo cuyos lados miden 36 m y 48 m respectivamente.

Base 48x : 12 = 4x

Altura 36x : 12 = 3x

(4x)2 + (3x)2 = 752

25x2 = 5625

X2 = 225 x = 15

Base 4 · 15 = 60 m

Altura 3 · 15 = 45 m

2. Dos números naturales se diferencian en dos unidades y la suma de sus cuadrados es 580. ¿Cuáles son esos números?

1er número x

2º número x + 2

3. Dos caños A y B llenan juntos una piscina en dos horas, A lo hace por sí solo en tres horas menos que B. ¿Cuántas horas tardas a cada uno separadamente?

Tiempo de A x

Page 108: repositorio.unesum.edu.ecrepositorio.unesum.edu.ec/bitstream/53000/2123/1/LIBRO-DIP-039.pdf · Quedan todos los derechos reservados. Esta publicación no puede ser reproducida, distribuida,

108

Tiempo de B x+ 3

A

B

A y B

Tiempo de A 3 horas

Tiempo de B 6 horas

Subsección 7..- Evaluación de la sección

4.24 Evaluación

Ejercicios resueltos:

1. Al calcular 2ª2 + 3a se obtiene:

___ 5ª2 ___ 5ª2

___ a (2a + 3) ___ Ninguna de las anteriores.

2. La expresión 2ª3 – 4a2 + 2a es equivalente a:

___ 2(a3 – 2ª2 + a) ___ 2a (a2 – 2a + 1)

___ a (2ª2 – 4a + 2) ___ 2a(a2 2a)

3. La expresión 4 a2 b – 2 a2 b2 + 6b3 se convierte extrayendo factor común en:

___ab (4a – 2ab + 6b2) ___2ab(2a2 a2b + 3b2)

___ 6ab(2ª2 +4 a2 b + b2) ___2 a 2b2

4. Rudy plantea que la siguiente expresión 25 x2 –10xy + 4 y2, se expresa como producto así: (5x – 2y)2. ¿Estás de acuerdo con Rudy? Fundamenta tu respuesta.

2a

b31

b

2

Page 109: repositorio.unesum.edu.ecrepositorio.unesum.edu.ec/bitstream/53000/2123/1/LIBRO-DIP-039.pdf · Quedan todos los derechos reservados. Esta publicación no puede ser reproducida, distribuida,

109

Solución:

Ejercicio 1: Repuesta correcta __X_ a (2a + 3)

Ejercicio 2: Repuesta correcta __X_ 2a (a2 – 2a + 1)

Ejercicio 3: Repuesta correcta __X_ a2 b2

Ejercicio 4: Si estoy de acuerdo con Rudy, ya que ((5x – 2y)2 = 25 x2 –10xy + 4 y2

5. Dados:

b) Calcule y simplifique A . B c) Halle el valor de x para el cual el valor numérico del resultado del inciso a) es -1.

Solución:

a)

b)

De donde se obtiene que: 3x = -6, o sea que: x = - 2

6. Sean:

Verifica que la expresión A. B + x2 se hace cero para un único valor de x

Solución:

;Al simplificar esta expresión se obtiene:

2x+1+x² y al comprobar para que valor de x esta expresión se hace cero se resuelve la ecuación: x² + 2x + 1 = 0; (x + 1)² = 0, la cual se hace cero para el único valor x = -1

2a

b31

b

2

xx

xxB

x

xxA

64

3612

36

18922

2

2

2

x

x

xx

x

xx

xx

xx

xx

x

xxBA

2

6

)32(2

)²6(

)6)(6(

)6)(32(

6²4

3612²

36²

189²2

xx

xx

x

26

2/12

6

42045

5112 2

23

2

xBy

xxx

xxA

²)2)(2()5)(2)(2(

)5)(12(²)4²(

204²5³

511²2² xxx

xxx

xxxx

xxx

xxxBA

Page 110: repositorio.unesum.edu.ecrepositorio.unesum.edu.ec/bitstream/53000/2123/1/LIBRO-DIP-039.pdf · Quedan todos los derechos reservados. Esta publicación no puede ser reproducida, distribuida,

110

7. Sea: 4-x

4+x3-x = A(x)

2

23

Determine el conjunto de los números reales no negativos para los cuales se cumple que A(x)0.

Solución:

04-x

4+x3-x = A(x)

2

23

0)2)(2(

)2)(1)(2(

xx

xxx Al buscar los ceros del numerador y el denominador se obtiene

Ceros del numerador: x1=2 (doble) y x2=-1

Ceros del denominador: x3 = 2 y x4 = -2

Solución gráfica:

Solución Analítica: S = [0; 2)

8. Resuelve la inecuación 2094

1

5 2

xx

x

xx

x

Solución:

2094

1

5 2

xx

x

xx

x

02094

1

5 2

xx

x

xx

x

0)4)(5(4

1

5

xx

x

xx

x

;

0)4)(5(

)5()4(

xx

xxxx

0)4)(5(

54²

xx

xxxx

;

0)4)(5(

56²

xx

xx

0)4)(5(

)1)(5(

xx

xx

Ceros del numerador: x1=1 y x2=5

Page 111: repositorio.unesum.edu.ecrepositorio.unesum.edu.ec/bitstream/53000/2123/1/LIBRO-DIP-039.pdf · Quedan todos los derechos reservados. Esta publicación no puede ser reproducida, distribuida,

111

Ceros del denominador: x3 = 5 y x4 = 4

Solución gráfica:

Solución Analítica: S = [1; 4)

9.

10.

11.

Page 112: repositorio.unesum.edu.ecrepositorio.unesum.edu.ec/bitstream/53000/2123/1/LIBRO-DIP-039.pdf · Quedan todos los derechos reservados. Esta publicación no puede ser reproducida, distribuida,

112

12.

15.

4.25 Ejercicios y problemas sobre Ecuaciones Cuadráticas

1. Una pieza rectangular es 4 cm más larga que ancha. Con ella se construye una caja de 840 cm3 cortando un cuadrado de 6 cm de lado en cada esquina y doblando los bordes. Halla las dimensiones de la caja.

6 (x − 12) · (x + 4 −12) = 840 (x − 12) · (x −8) = 140

X2 − 20x − 44 = 0 x = 22 y x= −2.

Las dimensiones son: 26 cm y 22 cm.

2. Un caño tarda dos horas más que otro en llenar un depósito y abriendo los dos juntos se llena en 1 hora y 20 minutos. ¿Cuánto tiempo tardará en llenarlo cada uno por separado?

Tiempo del 1º x

Page 113: repositorio.unesum.edu.ecrepositorio.unesum.edu.ec/bitstream/53000/2123/1/LIBRO-DIP-039.pdf · Quedan todos los derechos reservados. Esta publicación no puede ser reproducida, distribuida,

113

Tiempo de 2º x − 2

El primero llena un depósito en x horas.

El segundo llena un depósito en (x − 2) horas

1 hora y 20 minutos = 4/3 horas

Entre los dos

Entre los dos llenan un depósito en 4/3 horas

Tiempo del 1º 4 horas

Tiempo de 2º 2 horas

no es una solución, porque el tiempo empleado por el segundo caño sería negativo.

Resolver sistemas de ecuaciones aplicando el método de Gauss.

Page 114: repositorio.unesum.edu.ecrepositorio.unesum.edu.ec/bitstream/53000/2123/1/LIBRO-DIP-039.pdf · Quedan todos los derechos reservados. Esta publicación no puede ser reproducida, distribuida,

114

EJERCICIOS OPCIONALES (COMPLEMENTARIOS)

1. Expresa como producto:

1) 14 x2 y2 – 28 x3 +56 x4 2) 81 – y²

3) ay2 – 10ay + 25a 4) mx²+9mx+20m

5) 6) 2a2 – 8

7) 55 m2 n3 x + 110 m2 n3 x2 –220 m 2g2 8) (a – b) + (a – b)r

1x4

x 2

Page 115: repositorio.unesum.edu.ecrepositorio.unesum.edu.ec/bitstream/53000/2123/1/LIBRO-DIP-039.pdf · Quedan todos los derechos reservados. Esta publicación no puede ser reproducida, distribuida,

115

9) m2 +28m –165 10) 3a (p – q) + 2p – 2q

11) mx2 + 9xm + 20m 12) 6x2 – 11ax – 7a2

13) 14) 4x3 y + 10 x2 y2 – 24xy3

15) (a – b)2 – ( a – b)2 16) y2 + 40x + 25

17) 18) x³ - 4x² - 9x +36

19) 8x³ - 27 20) a³ + 64

21. Sean las expresiones: M = x³−7x+6

x³+8; N =

x³−9

x²−2x+4; P =

2x−2

x−1;

Hallar la expresión Q tal que: Q = M : N + P

Dadas las expresiones algebraicas siguientes:

A = a+5

a³−2a2+2a−4; B =

a²−3

12+4a²− a4; C = a−4

a³−2a2−6a+ 1

Hallar la expresión D tal que:

D = A – B – C

Sean las expresiones: O = 1

x−1, S =

1

x²+x−2 , A =

x+1

x³+4x²+x−6 ,

Hallar la expresión R, tal que: R = O + S + A

Efectúa las operaciones siguientes:

y+2

2x−

y4+6y2+9

y3−y2+3y−3∶

2xy²+6x

y²−1−2xy+2x

a² − 2a + 1

a ² − 1∶

a³ − 1

a³ − 4a2 − a + 4 ∙

a² + a + 1

a² − 9a + 20

aah3

2ah

3

2 2

4

1

42 vu36

1

Page 116: repositorio.unesum.edu.ecrepositorio.unesum.edu.ec/bitstream/53000/2123/1/LIBRO-DIP-039.pdf · Quedan todos los derechos reservados. Esta publicación no puede ser reproducida, distribuida,

116

Page 117: repositorio.unesum.edu.ecrepositorio.unesum.edu.ec/bitstream/53000/2123/1/LIBRO-DIP-039.pdf · Quedan todos los derechos reservados. Esta publicación no puede ser reproducida, distribuida,

117

REFERENCIAS BIBLIOGRÁFICAS

BIBLIOGRAFÍA BÁSICA

Amufan L. (2010). Matemática Intermedia. U.S. Editorial de Pearson 2012.

Espinoza, E. (2004). Álgebra pre universitaria. Volumen II. (1ª Edición.). Lima.

Espinoza, E. (2000). Números Complejos y Ecuaciones Polinómicas. (2ª ed.). Lima.

Grossman, S & Stanley, I. (2008). Álgebra Lineal. (6ª ed.). México. T

Kalnin, R.A. (1988). Algebra y funciones elementales. Editorial Mir. Moscú.

Kolman B. (1997). Estructuras de Matemáticas Discretas. Editorial Prentice-Hall

Kletenik, D. (1979). Problemas de Geometría Analítica. Moscú. Editorial Mir.

Lehmann, Ch. (1993). Geometría Analítica. México. Editorial Limusa S.A.

Paredes, A. (2006). Razonamiento Lógico – matemático. Editorial San Marcos.

Rodríguez, R. A. & Aliaga, C. S. (2015). Elementos de Matemática Básica para Carreras Universitarias. Libro digital contenido en el Soft Ware SUPERSOF.

Salinas, G. (2012). Álgebra Superior. (1ª ed.). Riobamba. Editorial Soluciones Gráficas

Yakovliev, G. N. (1982). Geometría. Editorial Mir. Moscú.

BIBLIOGRAFÍA COMPLEMENTARIA

B.C.1. Fundamentos de Matemática. Ecuador. Editorial ICM-ESPOL 2007 Código 9789978310311.

B.C.2. http://www.geocities.com/CapeCanaveral/Hangar/6374/dragon3.html.

B.C.3. http://www.etsimo.uniovi.es/usr/adolfo/algebra1.html.

Page 118: repositorio.unesum.edu.ecrepositorio.unesum.edu.ec/bitstream/53000/2123/1/LIBRO-DIP-039.pdf · Quedan todos los derechos reservados. Esta publicación no puede ser reproducida, distribuida,
Page 119: repositorio.unesum.edu.ecrepositorio.unesum.edu.ec/bitstream/53000/2123/1/LIBRO-DIP-039.pdf · Quedan todos los derechos reservados. Esta publicación no puede ser reproducida, distribuida,
Page 120: repositorio.unesum.edu.ecrepositorio.unesum.edu.ec/bitstream/53000/2123/1/LIBRO-DIP-039.pdf · Quedan todos los derechos reservados. Esta publicación no puede ser reproducida, distribuida,
Page 121: repositorio.unesum.edu.ecrepositorio.unesum.edu.ec/bitstream/53000/2123/1/LIBRO-DIP-039.pdf · Quedan todos los derechos reservados. Esta publicación no puede ser reproducida, distribuida,
Page 122: repositorio.unesum.edu.ecrepositorio.unesum.edu.ec/bitstream/53000/2123/1/LIBRO-DIP-039.pdf · Quedan todos los derechos reservados. Esta publicación no puede ser reproducida, distribuida,